You are on page 1of 218

CHAPTER 1 SOLUTIONS

(1-1)

(1-2)
25V

20V

15V

10V

5V

0V

-5V
0s 2us 4us 6us 8us 10us 12us 14us 16us
V(D1:2)
Time

25V

(1.4333u,23.800)
20V

15V

10V

5V

(4.0833u,-851.690m)

0V

-5V
0s 2us 4us 6us 8us 10us 12us 14us 16us
V(S1:4)
Time

In part (b), the voltage across the current source is reduced from 24 V by the switch resistance
and diode voltage drop.
(1-3)

40V

96.46n,23.94)

20V

0V (3.150u,-1.052)
(3.150u,-1.052)

-20V
0s 5us 10us 15us
V(V2:-)
Time

(1-4)
25V

(800.000n,23.924)
20V

15V

10V

5V

(3.8333u,-1.0517)

0V

-5V
0s 2us 4us 6us 8us 10us 12us 14us 16us
V(V2:-)
Time

CHAPTER 2 SOLUTIONS
2/21/10

2-1) Square waves and triangular waves for voltage and current are two examples.

_____________________________________________________________________________
________

v 2 ( t ) [170sin ( 377t )]2


2-2) a) p ( t ) = v ( t ) i ( t ) = = = 2890sin 2 377t W .
R 10
b) peak power = 2890 W.

c) P = 2890/2 = 1445 W.

_____________________________________________________________________________
________
2-3)

v(t) = 5sin2πt V.

a) 4sin2πt A.; p(t) = v(t)i(t) = 20 sin22πt W.; P = 10 W.

b) 3sin4πt A.; p(t) = 15sin(2πt)sin(4πt) W.; P = 0

_____________________________________________________________________________
________

2-4) a)

0 0  t  50 ms

p ( t ) = v ( t ) i ( t ) = 40 50 ms  t  70 ms
0 70 ms  t  100 ms

b)
T 70 ms
1 1
 v ( t ) i ( t ) dt =
100 ms 50ms
P= 40 dt = 8.0 W .
T 0
c)
T 70 ms

W =  p ( t ) dt =  40 dt = 800 mJ .; or W = PT = (8W )(100 ms ) = 800 mJ .


0 50 ms

_____________________________________________________________________________
________

2-5) a)

70 W . 0  t  6 ms
−50 W . 6 ms  t  10 ms

p (t ) = v (t ) i (t ) = 
40 W . 10 ms  t  14 ms
0 14 ms  t  20 ms

b)

1  
T 6 ms 10 ms 14 ms
1
P=  p ( t ) dt =   70 dt +  ( −50 ) dt +  40 dt  = 19 W .
T 0 20 ms  0 6 ms 10 ms 

c)
T  6 ms 10 ms 14 ms

W =  p ( t ) dt =   70 dt +  ( −50 ) dt +  40 dt  = 0.38 J .;
0  0 6 ms 10 ms 
or W = PT = (19 )( 20 ms ) = 380 mJ .
_____________________________________________________________________________
________

2-6)

P = Vdc I avg
a ) I avg = 2 A., P = (12 )( 2 ) = 24 W .
b) I avg = 3.1 A., P = (12 )( 3.1) = 37.2 W .
_____________________________________________________________________________
________

2-7)

a)

vR ( t ) = i ( t ) R = 25sin 377t V .
p ( t ) = v ( t ) i ( t ) = ( 25sin 377t )(1.0sin 377t ) = 25sin 2 377t = 12.5 (1 − cos 754t ) W .
T
1
p ( t ) dt = 12.5 W .
T 0
PR =

b)

di ( t )
vL ( t ) = L = 10 (10 ) ( 377 )(1.0 ) cos 377t = 3.77 cos 377t V .
−3

dt

pL ( t ) = v ( t ) i ( t ) = ( 3.77 cos 377t )(1.0sin 377t ) =


( 3.77 )(1.0 ) sin 754t = 1.89sin 754t W .
2
T
1
PL =  p ( t ) dt = 0
T 0

c)

p ( t ) = v ( t ) i ( t ) = (12 )(1.0sin 377t ) = 12sin 377t W .


T
1
p ( t ) dt = 0
T 0
Pdc =

_____________________________________________________________________________
________
2-8) Resistor:

v ( t ) = i ( t ) R = 8 + 24sin 2 60t V .
p ( t ) = v ( t ) i ( t ) = ( 8 + 24sin 2 60t )( 2 + 6sin 2 60t )
= 16 + 96sin 2 60t + 144sin 2 2 60t W .
1  
T 1/60 1/60 1/60
1
P=  p ()
t dt =  16 dt +  96sin 2 60t dt +  144sin 2 2 60t 
T 0 1/ 60  0 0 0 
= 16 + 72 = 88 W .
Inductor: PL = 0.

dc source: Pdc = I avgVdc = ( 2 )( 6 ) = 12 W .

_____________________________________________________________________________
________

2-9) a) With the heater on,

P=
Vm I m
= 1500 W . → I m =
(1500 )( 2 ) = 12.5 2
2 120 2
( )( )
p ( t ) = Vm I m sin 2t = 120 2 12.5 2 sin 2t = 3000sin 2t
max ( p ( t ) ) = 3000 W .
b) P = 1500(5/12) = 625 W.

c) W = PT = (625 W)(12 s) = 7500 J. (or 1500(5) = 7500 W.)

_____________________________________________________________________________
________

2-10)
t
1 1
iL ( t ) =  vL ( t ) dt =
0.1 0
90 d  = 900t 0  t  4 ms.
L
iL ( 4 ms ) = ( 900 )( 4 )(10 ) = 3.6 A.
−3

a)

1 1
W = Li 2 = ( 0.1)( 3.6 ) = 0.648 J .
2

2 2

b) All stored energy is absorbed by R: WR = 0.648 J.


c)

WR 0.648
PR = = = 16.2 W .
T 40 ms
PS = PR = 16.2 W .

d) No change in power supplied by the source: 16.2 W.

_____________________________________________________________________________
________
2-11)

a)

1 2 2W 2 (1.2 )
W= Li , or i = = = 15.49 A.
2 L 0.010
t t
1 1
i (t ) =  v ( ) d =
0.010 0
14 d  = 1400t A.
L0
15.49 = 1400ton
ton = 11.1 ms
b) Energy stored in L must be transferred to the resistor in (20 - 11.1) = 8.9 ms.
Allowing five time constants,

L 8.9 ms L 10 mH
 = = 1.7 ms.; R = = 5.62 
R 5 1.7 ms 1.7 ms

_____________________________________________________________________________
________

2-12)

a) i(t) = 1800t for 0 < t < 4 ms

i(4 ms) = 7.2 A.; WLpeak = 1.296 J.

b)
10A

5A Inductor current
SEL>>
0A
I(L1)
10A

Source current
0A

-10A
-I(Vcc)
1.0KW
Ind. inst. power
0W

-1.0KW
W(L1)
1.0KW
Source inst. power (supplied)
0W

-1.0KW
0s 20ms 40ms 60ms 80ms 100ms
-W(Vcc)
Time

_____________________________________________________________________________
________
2-13)

a) The zener diode breaks down when the transistor turns off to maintain inductor
current.

b) Switch closed: 0 < t < 20 ms.

diL ( t )
vL = 12 V . = L
dt
diL vL 12
= = = 160 A/s
dt L 0.075
at t = 20 ms, iL = (160 )( 0.02 ) = 3.2 A.

Switch open, zener on:

vL = 12 − 20 = −8 V .
diL vL −8
= = = −106.7 A/s
dt L 0.075
t to return to zero :
i −3.2
t = = = 30 ms
−106.7 −106.7
Therefore, inductor current returns to zero at 20 + 30 = 50 ms.

iL = 0 for 50 ms < t < 70 ms.

c)
40mW

Inductor inst. power


0W

-40mW
W(L1)
80mW

Zener inst. power

40mW

SEL>>
0W
0s 10ms 20ms 30ms 40ms 50ms 60ms 70ms
W(D1)
Time

d)

PL = 0.
1 1
( 0.03)( 64 ) = 13.73 W .
T
1
PZ =  pZ ( t ) dt = 
T 0 0.07  2 
_____________________________________________________________________________
________

2-14) a) The zener diode breaks down when the transistor turns off to maintain inductor
current.

b) Switch closed: 0 < t < 15 ms.

diL( t )
vL = 20 V . = L
dt
diL vL 20
= = = 400 A/s
dt L 0.050
at t = 15 ms, iL = ( 400 )( 0.015 ) = 6.0 A.
Switch open, zener on:

vL = 20 − 30 = −10 V.
diL vL −10
= = = −200 A/s
dt L 0.050
t to return to zero :
i −6.0
t = = = 30 ms
−200 −200
Therefore, inductor current returns to zero at 15 + 30 = 45 ms.

iL = 0 for 45 ms < t < 75 ms.

c)
200W

Inductor inst. power

0W

-200W
W(L1)
200W

Zener inst. power

100W

SEL>>
0W
0s 20ms 40ms 60ms 80ms
W(D1)
Time

d)

PL = 0.
1 1
( 0.03)(180 ) = 36 W .
T
1
PZ =  pZ ( t ) dt = 
T0 0.075  2 
_____________________________________________________________________________
________

2-15) Examples are square wave (Vrms = Vm) and a triangular wave (Vrms = Vm/√3).

_____________________________________________________________________________
________
Phase conductors: P = I R = 12 ( 0.5 ) = 72 W .
2 2
2-16)

( ) ( 0.5) = 216 W .
2
Neutral conductor: PN = I R = 12 3
2

Ptotal = 3 ( 72 ) + 216 = 432 W .


PN 72
RN = = = 0.167 
( )
2 2
IN 12 3

_____________________________________________________________________________
________

2-17) Re: Prob. 2-4

Vrms = Vm D = 10 0.7 = 8.37 V .


I rms = I m D = 4 0.5 = 2.83 A.

_____________________________________________________________________________
________

2-18) Re: Prob. 2-5

 14 
Vrms = Vm D = 10   = 8.36 V .
 20 
0.006 0.01 0.02
1
=  7 dt +  ( −5) dt + 4 dt = 27.7 = 5.26 A.
2 2 2
I rms
0.02 0 0.006 0.01

_____________________________________________________________________________
________
2-19)

2 2
 5   3 
Vrms = 22 +   +  = 4.58 V .
 2  2
2 2
 2   1.1 
I rms = 1.5 + 
2
 +  = 2.2 A.
 2  2

V I
P = V0 I 0 +  m m cos ( n − n )
n =1 2
 5  2   3  1.1 
= ( 2.0 )(1.5 ) +    cos ( −20 ) +    cos ( −115 ) = 7.0 W .
 2  2   2  2 
Note that − cos(4 60t + 45) is cos ( 4 60t − 135 )
_____________________________________________________________________________
________

2-20)

dc : V0 = 3 (100 ) = 300 V .
1 = 2 60 : Y1 = 1/R + jC = 0.01 + j 0.0189
I1 40
V1 = = = 187 − 62.1
Y1 ( 0.01 + j 0.0189 )
2 = 4 60 : Y2 = 1/R + jC = 0.01 + j 0.0377
I2 60
V2 = = = 153 − 75.1
Y2 ( 0.01 + j 0.0377 )

Vm I m
P = V0 I 0 +  cos ( n − n )
n =1 2

= 300 ( 5 ) +
(187 )( 4 ) cos (153)( 6 ) cos
( 62.1 ) + ( 75.1 )
2 2
= 1500 + 175 + 118 = 1793 W .

_____________________________________________________________________________
________

2-21) dc Source:

 50 − 12 
Pdc = Vdc I avg = 12  = 114 W .
 4 
Resistor:

P = I rms
2
R
I rms = I 02 + I1,2rms + I 2,2 rms
I 0 = 9.5 A.
30
I1 = = 3.51 A.
4 + j ( 4 60 )( 0.01)
10
I2 = = 0.641 A.
4 + j ( 8 60 )( 0.01)
2 2
 3.51   0.641 
I rms = 9.5 +  2
 +  = 9.83 A.
 2   2 
PR = I rms
2
R = 386 W .
_____________________________________________________________________________
________

2-22)

P = I rms
2
R
V0 6
I0 = = = 0.375 A.
R 16
5
I1 = = 0.269 A.
16 + j ( 2 60 )( 0.025 )
3
I2 = = 0.0923 A.
16 + j ( 6 60 )( 0.025 )
2 2
 0.269   0.0923 
I rms = 0.375 +  2
 +  = 0.426 A.
 2   2 
I rms = 0.623 A.; P = I rms R = ( 0.426 ) (16 ) = 2.9 W .
2 2

_____________________________________________________________________________
________

2-23)

Vm I m
P = V0 I 0 +  cos ( n − n )
n =1 2
n Vn In Pn ∑Pn

0 20 5 100 100

1 20 5 50 150
2 10 1.25 6.25 156.25

3 6.67 0.556 1.85 158.1

4 5 0.3125 0.781 158.9

Power including terms through n = 4 is 158.9 watts.

_____________________________________________________________________________
________

2-24)

Vm I m
P = V0 I 0 +  cos ( n − n )
n =1 2
n Vn In θn - ϕn° Pn

0 50.0000 10.0 0 500.0

1 50.0000 10.0 26.6 223.6

2 25.0000 2.5 45.0 22.1

3 16.6667 1.11 56.3 5.1

4 12.5000 0.625 63.4 1.7

Through n = 4, ∑Pn = 753 W.

_____________________________________________________________________________
________

2-25)

Vm I m
P = V0 I 0 +  cos ( n − n )
n =1 2
V −V 50 − 36
I 0 = 0 dc = = 0.7 A
R 20
P0, R = I 02 R = ( 0.7 ) 20 = 9.8 W (dc component only )
2

PVdc = I 0Vdc = ( 0.7 )( 36 ) = 25.2 W


PL = 0
Resistor Average Power

n Vn Zn In angle Pn
0 50.00 20.00 0.7 0.00 9.8

1 127.32 25.43 5.01 0.67 250.66

2 63.66 37.24 1.71 1.00 29.22

3 42.44 51.16 0.83 1.17 6.87

4 31.83 65.94 0.48 1.26 2.33

5 25.46 81.05 0.31 1.32 0.99

PR = ∑ Pn ≈ 300 W.

_____________________________________________________________________________
________

2-26) a) THD = 5% → I9 = (0.05)(10) = 0.5 A.

b) THD = 10% → I9 = (0.10)(10) = 1 A.

c) THD = 20% → I9 = (0.20)(10) = 2 A.

d) THD = 40% → I9 = (0.40)(10) = 4 A.

_____________________________________________________________________________
________

2-27) a)

 170  10 
P =  Pn =    cos ( 30 ) + 0 + 0 = 736 W .
 2  2 

b)

2 2 2
 10   6   3 
I rms =   +  +  = 8.51 A.
 2  2  2
 170 
S = Vrms I rms =   8.51 = 1024 VA.
 2
P 736
pf = = = 0.719
S 1024
c)

I1,rms 10/ 2
DF = = = 0.831
I rms 8.51

d)

2 2
 6   3 
  + 
 2  2
THDI = = 0.67 = 67%
10/ 2
_____________________________________________________________________________
________

2-28) a)

 170  12 
P =  Pn =    cos ( 40 ) + 0 + 0 = 781 W .
 2  2 
b)

2 2 2
 12   5   4 
I rms =   +  +  = 9.62 A.
 2  2  2
 170 
S = Vrms I rms =   9.62 = 1156 VA.
 2
P 781
pf = = = 0.68
S 1156
c)

I1,rms 12/ 2
DF = = = 0.88
I rms 9.62
d)

2 2
 5   4 
  + 
 2  2
THDI = = 0.53 = 53%
12/ 2
_____________________________________________________________________________
________

2-29)
8 4
I1,rms = = 5.66 A.; I 2,rms = = 2.82 A.;
2 2
I rms = 5.662 + 2.822 = 6.32 A.; I peak  10.38 ( graphically)

a) P = V1,rms I1,rms cos (1 − 1 ) = ( 240 )( 5.66 ) cos ( 0 ) = 1358 W .

P P 1358
b) pf = = = = 0.895 = 89.5%
S Vrms I rms ( 240 )( 6.32 )

I 2,rms 2.82
c) THDI = = = 0.446 = 44.6%
I rms 6.32

I1,rms 5.66
d) DF = = = 89.6%
I rms 6.32

I peak 10.38
e) crest factor = = = 1.64
I rms 6.32

_____________________________________________________________________________
________

2-30)

12 9
I1,rms = = 8.49 A.; I 2,rms = = 6.36 A.;
2 2
I rms = 8.492 + 6.362 = 10.6 A.; I peak  18.3 A. ( graphically)

a) P = V1,rms I1,rms cos (1 − n ) = ( 240 )(10.6 ) cos ( 0 ) = 2036 W .

P P 2036
b) pf = = = = 0.80 = 80%
S Vrms I rms ( 240 )(10.6 )

I 2,rms 6.36
c) THDI = = = 0.60 = 60%
I rms 10.6
I1,rms 8.49
d) DF = = = 80%
I rms 10.6

I peak 18.3
e) crest factor = = = 1.72
I rms 10.6

_____________________________________________________________________________
________
2-31)

5V: I = 0 (capacitor is an open circuit)

1 1
25cos(1000t ): Z = R + j L − j = 2 + j1000(.001) − j = 2 + j0
C 1000 (1000 )10−6
25
I= cos(1000t ) = 12.5cos(1000t ) A
2
10cos(2000t ): Z = 2 + j1.5 
10
I10 = = 4 − 37 A.
2 + j1.5
2 2
 12.5   4 
I rms =   +  = 9.28 A
 2   2

PR = I rms
2
R = 9.282 ( 2 ) = 172.3 W

PL = 0

PC = 0

Psource = −172.3 W

_____________________________________________________________________________
________

2-32) PSpice shows that average power is 60 W and energy is 1.2 J. Use VPULSE and IPULSE for
the sources.
Energy
2.0 (20.000m,1.2000)

0
S(W(I1))
400W
Avg Power (20.000m,60.000)

0W
Inst Power

-400W
W(I1) AVG(W(I1))
20

SEL>>
-20
0s 4ms 8ms 12ms 16ms 20ms
I(I1) V(V1:+)
Time

_____________________________________________________________________________
________
2-33) Average power for the resistor is approximately 1000 W. For the inductor and dc source,
the average power is zero (slightly different because of numerical solution).

2.0KW
Average Power

(16.670m,0.9998K)
Resistor
1.0KW

Inductor
(16.670m,-30.131u)
0W
(16.670m,189.361u)
Vdc

-1.0KW
0s 5ms 10ms 15ms 20ms
AVG(W(R1)) AVG(W(L1)) AVG(W(V1))
Time
2.0KW
Instantaneous Power

Resistor

1.0KW

Inductor

0W

Vdc

-1.0KW
0s 5ms 10ms 15ms 20ms
W(R1) W(L1) W(V1)
Time

_____________________________________________________________________________
________
2-34)
Rms voltage is 8.3666 V. Rms current is 5.2631 A.

10V

Voltage
(20.000m,8.3666)
5V

0V
V(V1:+) RMS(V(V1:+))
10A
(20.000m,5.2631)

Current
0A

SEL>>
-10A
0s 4ms 8ms 12ms 16ms 20ms
I(I1:+) RMS(I(I1))
Time

_____________________________________________________________________________
________

2-35) See Problem 2-10.


0W

(40.022m,-16.200)
Source Power
-100W

SEL>>
-200W
AVG(W(V1))
4.0

Inductor

2.0
(4.0000m,648.007m)
Resistor
(40.021m,647.946

0
0s 10ms 20ms 30ms 40ms
I(L1) S(W(L1)) S(W(R1))
Time

The inductor peak energy is 649 mJ, matching the resistor absorbed energy. The source power
is -16.2 W absorbed, meaning 16.2 W supplied.

b) If the diode and switch parameters are changed, the inductor peak energy is 635 mJ, and
the resistor absorbed energy is 620 mJ. The difference is absorbed by the switch and diode.

_____________________________________________________________________________
________
2-36)

The inductor current reaches a maximum value of 3.4 A with the resistances in the circuit: I =
75/(20+1+1) = 3.4 A.
4.0A

2.0A Inductor Current

SEL>>
0A
I(L1)
4.0A

Source Current

0A

-4.0A
0s 20ms 40ms 60ms 80ms 100ms
-I(V1)
Time

Quantity Probe Expression Result

Inductor resistor average AVG(W(R1)) 77.1 W


power

Switch average power AVG(W(S1)) 3.86 W each

Diode average power AVG(W(D1)) 81 mW each

Source average power AVG(W(Vcc)) -85.0 W


_____________________________________________________________________________
________

2-37)

a) Power absorbed by the inductor is zero. Power absorbed by the Zener diode is 13.8 W.

4.0A

2.0A
Inductor Current

0A
I(L1)
4.0A

2.0A Zener Diode Current

SEL>>
0A
0s 10ms 20ms 30ms 40ms 50ms 60ms 70ms
-I(D1)
Time
b) Power in the inductor is zero, but power in the 1.5Ω resistor is 1.76 W. Power absorbed by
the Zener diode is 6.35 W. Power absorbed by the switch is 333 mW.

_____________________________________________________________________________
________
3-38)

See Problem 3-37 for the circuit diagram.

a) Power absorbed by the Zener diode is 36.1 W. Power absorbed by the inductor is zero.

10A

5A
Inductor Current

SEL>>
0A
I(L1)
10A

5A Zener Diode Current

0A
0s 20ms 40ms 60ms 80ms
-I(D1)
Time

b) Power in the inductor is zero, but power in the 1.5Ω resistor is 4.4 W. Power absorbed by
the Zener diode is 14.2 W. Power absorbed by the switch is 784 mW.
2-39)

40A

Total Current

20A

0A

-20A
0s 4ms 8ms 12ms 16ms 20ms
I(I1) I(I2) I(I3) I(I4) -I(V1)
Time

Quantity Probe Expression Result

Power AVG(W(V1)) 650 W


rms current RMS(I(V1)) 14 A

Apparent power S RMS(V(I1:+))* RMS(I(V1)) 990 VA

Power factor AVG(W(V1)) / (RMS(V(I1:+))* RMS(I(V1))) 0.66

_____________________________________________________________________________
________

2-40)

DESIRED QUANTITY ORIGINAL RESULT NEW VALUES

Inductor Current max = 4.5 A. 4.39 A

Energy Stored in Inductor max = 2.025 J 1.93 L

Average Switch Power 0.010 W. 0.66 W

Average Source Power (absorbed) -20.3 W. -19.9 W

Average Diode Power 0.464 W. .449 W


AVG(W(D1))
0.464 W.
Average Inductor Power 0 0

Average Inductor Voltage 0 0

Average Resistor Power 19.9 W. 18.8 W

Energy Absorbed by Resistor 1.99 J. 1.88 J

Energy Absorbed by Diode .046 J. .045 J

Energy Absorbed by Inductor 0 0

rms Resistor Current 0.998 A. 0.970 A

_____________________________________________________________________________
________
2-41) Use the part VPULSE or IPULSE (shown). Here, the period is 100 ms, and the rise times
chosen are 20 ms, 50 ms, and 80 ms. The fall times are the period minus the rise times. Each
rms value is 0.57735, which is identical to 1/√3.

1.0A

(100.000m,577.350m)

0A

-1.0A
0s 20ms 40ms 60ms 80ms 100ms
-I(R1) RMS(I(R1))
Time

_____________________________________________________________________________
________

CHAPTER 3 SOLUTIONS
2/20/10

3-1)
V0 Vm 170 / 
a) I 0 = = = = 3.60 A.
R R 15
V V 170
b) I rms = rms = m = = 5.66 A.
R 2 R 2(15)
c) P = I 2 R = 5.662 (15) = 480 W .
 170 
d ) S = Vrms I rms =   (5.66) = 679 VA.
 2
P 480 W
e) pf = = = 0.707 = 70.7%
S 679 VA

3-2)
V0
a ) I 0 = 12 A.; I 0 = → V0 = I 0 R = (12)(20) = 240 V .
R
Vm
Vo = ; Vm = Vo = 240  = 754 V .

754
Vrms = = 533 V .
2
N1 240
= = 0.45
N 2 533
N2 12
b) I o  = I o = = 26.7 A.
N1 0.45

3-3)
P P V V V
a) pf = = ; I rms = s ,rms ; VR ,rms = m ; Vs ,rms = m
S Vs ,rms I rms R 2 2
 Vm 
2  / R
pf =
V /R
R , rms
=  2  =
2
=
1
Vs ,rms I rms  Vm  Vm  2 2
  2  / R
 2  
b) Displacement pf = cos(1 −  ) = cos(0) = 1
V1 1 Vm 1
I1 = = 0; pf = cos(1 − 1 ) DF ;  DF =
R R 2 2

3-4) Using Eq. 3-15,


Vm V
a) i (t ) = sin(t −  ) + m (sin  )e −t /
Z Z
Z = R 2 + ( L) 2 = 122 + (377(0.012))2 = 12.8 
 L  −1  377(0.012) 
 = tan −1   = tan   = 0.361 rad
 R   12 
 L 377(0.012)
 = = = 0.377
R 12
i (t ) = 13.2sin(t − 0.361) + 4.67e −t /0.377 :  = 3.50 rad = 201
b) I avg = 4.36 A. (numerical integration)
c) I rms = 6.70 A. (numerical integration) P = I rms
2
R = (6.70) 2 (12) = 538 W .
P 538
d ) pf = = = 0.67
S (120)(6.70)

3-5) Using Eq. 3-15,


Vm V
a ) i (t ) = sin(t −  ) + m (sin  )e −t /
Z Z
Z = R 2 + ( L) 2 = 102 + (377(0.015))2 = 11.5 
 L  −1  377(0.015) 
 = tan −1   = tan   = 0.515 rad
 R   10 
 L 377(0.015)
 = = = 0.565
R 10
i (t ) = 14.8sin(t − 0.515) + 7.27e −t /0.565 :  = 3.657 rad = 209.5
b) I avg = 5.05 A. (numerical integration)
c) I rms = 7.65 A. (numerical integration) P = I rms
2
R = (7.65) 2 (10) = 584 W .
P 584
d ) pf = = = 0.637 = 63.7%
S (120)(7.65)
3-6) Using Eq. 3-15,

Vm V
a ) i (t ) = sin(t −  ) + m (sin  )e −t /
Z Z
Z = R 2 + ( L) 2 = 152 + (377(0.08)) 2 = 33.7 
 L  −1  377(0.08) 
 = tan −1   = tan   = 1.11 rad
 R   15 
 L 377(0.08)
 = = = 2.01
R 15
i (t ) = 10.1sin(t − 1.11) + 9.02e −t /2.01 :  = 4.35 rad = 250
b) I avg = 4.87 A. (numerical integration)
c) I rms = 6.84 A. (numerical integration) P = I rms
2
R = (6.84) 2 (15) = 701 W .
P 701
d ) pf = = = 0.427 = 42.7%
S (240)(6.84)

3-7) Using an ideal diode model, R = 48 Ω for an average current of 2 A.


8.0A

Current
Iavg = 2 A for R = 48 ohms
4.0A

(16.700m,2.0030)

Average Current

0A
0s 5ms 10ms 15ms 20ms
I(R1) AVG(I(L1))
Time

3-8) Using Eqs. 3-22 and 3-23,


Vm V
a) i (t ) = sin(t −  ) − dc + Ae −t /
Z R
 V V 
A =  − m sin( −  ) + dc  e /
 Z R 
Z = R 2 + ( L) 2 = 102 + (377(.075) 2 = 30.0 
 L  −1  377(.075) 
 = tan −1   = tan   = 1.23 rad
 R   10 
 L 377(0.075)
 = = = 2.83
R 10
V 100
 = sin −1 dc = = 0.299 rad = 17.1
Vm 240 2
i (t ) = 11.3sin(t − 1.23) − 10 + 21.2e −t /2.83 ;  = 3.94 rad = 226
I avg = 3.13 A. (numerical integration), Pdc = Vdc I avg = (100)(3.13) = 313 W .
b) I rms = 4.81 A. (numerical integration) PR = I rms
2
R = (4.81) 2 (10) = 231 W .
P 313 + 231
c) pf = = = 0.472 = 47.2%
S (240)(4.81)
3-9) Using Eqs. 3-22 and 3-23,
Vm V
a) i (t ) = sin(t −  ) − dc + Ae −t /
Z R
 V V 
A =  − m sin( −  ) + dc  e /
 Z R 
Z = R 2 + ( L) 2 = 122 + (377(0.12) 2 = 46.8 
 L  −1  377(0.12) 
 = tan −1   = tan   = 1.31 rad
 R   12 
 L 377(0.12)
 = = = 3.77
R 12
V 48
 = sin −1 dc = = 0.287 rad = 16.4
Vm 120 2
i (t ) = 3.63sin(t − 1.31) − 4.0 + 7.66e −t /3.77 ;  = 4.06 rad = 233
I avg = 1.124 A. (numerical integration), Pdc = Vdc I avg = (48)(1.124) = 54.0 W .
b) I rms = 1.70 A. (numerical integration) PR = I rms
2
R = (1.70) 2 (12) = 34.5 W .
P 54.0 + 34.5
c) pf = = = 0.435 = 43.5%
S (120)(1.70)

3-10) Using Eq. 3-33,


V V
i (t ) = m (cos  − cos t ) + dc ( − t )
L L
V   48 
 = sin −1  dc  = sin −1   = 0.287 rad .
 Vm   120 2 
i (t ) = 4.68 − 4.50 cos(t ) − 1.23t A.;  = 4.483 rad = 257
1 
2 
Io = i (t )d (t ) = 2.00 A.; Pdc = I oVdc = 2.00(48) = 96 W .
3-11)

300W

200W
L = 0.25 H

100W

0W
0s 5ms 10ms 15ms 20ms
AVG(W(Vdc))
Time

3-12) L ≈ 0.14 H for 50 W (51 W).


100W

(16.670m,51.156)
50W

L = 0.14 H

0W
0s 5ms 10ms 15ms 20ms
AVG(W(Vdc))
Time

3-13) Using Eq. 3-34,


a)

Vm 120 2 V0 54
V0 = = = 54.0 V .; I 0 = = = 4.50 A.
  R 12
b)
n Vn Zn In
0 54.02 12.00 4.50
1 84.85 25.6 3.31
2 36.01 46.8 0.77
4 7.20 91.3 0.08
The terms beyond n = 1 are insignificant.

3-14)

Run a transient response long enough to achieve steady-state results (e.g., 1000ms). The
peak-to-peak load current is approximately 1.48 A, somewhat larger than the 1.35 A
obtained using only the first harmonic. (The inductance should be slightly larger, about
0.7 H, to compensate for the approximation of the calculation.)

3-15)
a)
Vm 50
I0 = = = 3.98 A.
 R 4
V Vm / 2 25
I1 = 1 = = = 0.05I 0 = 0.199 A.
Z1 R 2 + ( L) 2 R 2 + ( L) 2
25
R 2 + ( L) 2 = 9 + ( L) 2 = = 125    L
0.199
125
L= = 0.33 H
2 60
b) A PSpice simulation using an ideal diode model gives 0.443 A p-p in the steady
state. This compares with 2(I1)=2(0.199)=0.398 A p-p.

3-16)
Vm 170
a ) V0 = = = 54.1 V
 
V0 − Vdc 54.1 − 24
I0 = = = 3.01 A.
R 10
io  1 A.  2 I1 → I1 = 0.5 A.
Vm 170
V1 = = = 85 V
2 2
V 85
Z1 = 1 = = 170  = R 2 + ( L) 2   L
I1 0.5
170
L= = 450 mH .
377
b) Pdc = I avgVdc = (3.01)(24) = 72.2 W .

c) PR = I rms
2
R; I rms = I 2
n , rms  (3.01) 2 + (0.5 / 2) 2 = 3.12 A.
PR = (3.12) 2 (10) = 97.4 W .

3-17) a) τ = RC = 10310-3=1 s; τ/T = 60. With τ >> T, the exponential decay is very
small and the output voltage has little variation.
b) Exact equations:
 = − tan −1 ( RC ) +  = − tan −1 (377) +  = 1.5573 rad = 90.15
Vm sin  = 200sin(90.15) = 199.9993
sin  − sin  e − (2 + + )/ RC = 0 →  = 1.391 rad = 79.72
Vo = Vm (1 − sin  ) = 3.21 V .
c) Approximation of Eq. 3-51:
Vm 200
Vo  = = 3.33 V .
fRC (60)(103 )(10−3 )
3-18) a) R = 100 Ω: τ = RC (100)10-3 = 0.1 s; τ/T = 6.

 = − tan −1 ( RC ) +  = − tan −1 (37.7) +  = 1.5973 rad = 91.52


Vm sin  = 200sin(91.52) = 199.93
sin  − sin  e − (2 + + )/ RC ) = 0 →  = 1.0338 rad = 59.23
Vo = Vm (1 − sin  ) = 28.16 V . (exact )
Vm 200
Vo  = = 33.3 V . (approximation)
fRC (60)(100)(10−3 )

b) R = 10 Ω: τ = RC (10)10-3 = 0.01 s; τ/T = .6.

 = − tan −1 ( RC ) +  = − tan −1 (3.77) +  = 1.830 rad = 104.9


Vm sin  = 200sin(104.9) = 193.3
sin  − sin  e − (2 + + )/ RC ) = 0 →  = 0.2883 rad = 16.5
Vo = Vm (1 − sin  ) = 143.2 V . (exact )
Vm 200
Vo  = = 333 V . (approximation)
fRC (60)(10)(10−3 )
In (a) with τ/T=6, the approximation is much more reasonable than (b) where
τ/T=0.6.

3-19) a) With C = 4000 µF, RC = 4 s., and the approximation of Eq. 3-51 should be
reasonable.

Vm 120 2
Vo  = = 0.707 V .
fRC (60)(4)
b) With C = 20 µF, RC = 0.02, which is on the order of one source period.
Therefore, the approximation will not be reasonable and exact equations must be
used.

 = − tan −1 ( RC ) +  = − tan −1 ((377)(1000)(20(10) −6 ) +  = 1.703 rad = 97.6)


 = 0.5324 rad = 30.5 (numerically from Eq. 3 − 43)
Vo = Vm − Vm sin  = 83.6 V .
3-20) a) With C = 4000 µF, RC = 2 s., and the approximation of Eq. 3-51 should be
reasonable.

Vm 120 2
Vo  = = 1.41 V .
fRC (60)(2.0)
b) With C = 20 µF, RC = 0.01, which is on the order of one source period.
Therefore, the approximation will not be reasonable and exact equations must be
used.

 = − tan −1 ( RC ) +  = − tan −1 ((377)(500)(20(10) −6 ) +  = 1.83 rad = 104.9)


 = 0.2883 rad = 16.5 (numerically from Eq. 3 − 43)
Vo = Vm − Vm sin  = 121 V .
3-21) From Eq. 3-51

Vm 120 2
C= = = 1,886  F
fRVo 60(750)(2)
 Vo  −1  2 
  sin −1 1 −  = sin 1 −  = 1.417 rad = 81.2
 Vm   120 2 
 sin  
I D , peak = Vm  C cos  +  = 18.7 A.
 R 
V
I D ,avg  m = 0.226 A.
R

3-22) Assuming Vo is constant and equal to Vm,

Vo2 Vm2 Vm2 (120 2) 2


P  → R= = = 576 
R R P 50
From Eq. 3-51

Vm 120 2
C= = = 3, 270  F
fRVo 60(576)(1.5)
 Vo  −1  1.5 
  sin −1 1 −  = sin 1 −  = 1.438 rad = 82.4
 Vm   120 2 
 sin  
I D , peak = Vm  C cos  +  = 28.1 A.
 R 
V
I D ,avg = m = 0.295 A.
R
3-23) Using the definition of power factor and Vrms from Eq. 3-53,
2 2
P Vrms /R Vrms /R V
pf = = = = rms
S (Vs ,rms )( I s ,rms ) (Vs ,rms )(Vrms / R) Vs ,rms
Vm  sin 2
1− +
= 2  2 = 1 1 −  + sin 2 = 1 −  + sin 2
Vm / 2 2  2 2 2 4
3-24)

Vm 120 2
a ) Vo = (1 + cos  ) = (1 + cos 45) = 46.1 V .
2 2
V2 V  sin 2
b) P = rms ; Vrms = m 1 − +
R 2  2
120 2 0.785 sin(2(0.785))
= 1− + = 80.9 V .
2  2
80.92
P= = 65.5 W .
100
 80.9  P 65.5
c) S = Vs ,rms I rms = (120)   = 97.1 VA; pf = = = 0.674 = 67.4%
 100  S 97.1

3-25)
Vm
a) vo = I o R = (2.5)(30) = 75 V = (1 + cos  )
2
 2 Vo   2 (75) 
 = cos −1  − 1 = cos −1  − 1 = 65.5 or 1.143 rad
 Vm   240 2 
Vo2,rms
b) P =
R
Vm  sin 2 240 2 1.143 sin(2(1.143))
Vo ,rms = 1− + = 1− + = 147.6 V .
2  2 2  2
147.62
P= = 726 W .
30
 147.6  P 726
c) S = Vs ,rms I rms = (240)   = 1181 VA; pf = = = 0.615 = 61.5%
 30  S 1181
3-26)
a) i (t ) = 5.42sin(t − 0.646) + 1.33e −t /0.754 A.
 = 25 = 0.524 rad ,  = 3.79 rad = 217 (numerically )

1
2 
b) I o = i (t )d (t ) = 1.80 A.


1 2
c) I rms =  i (t )d (t ) = 2.80 A.; Po = PR = I rms R = (2.80) 2 25 = 193 W .
2

2 
3-27)

a) i (t ) = 3.46sin(t − 0.615) − 6.38e −t /0.707 A.


 = 60 = 1.047 rad ,  = 3.748 rad = 215 (numerically)

1
2 
b) I o = i (t )d (t ) = 0.893 A.


1 2
=  i (t )d (t ) = 1.50 A.; Po = PR = I rms R = (1.50) 2 40 = 90.3 W .
2
c) I rms
2 

3-28) α ≈ 46°. Do a parametric sweep for alpha. Use the default (Dbreak) diode, and
use Ron = 0.01 for the switch. Alpha of 46 degrees results in approximately 2 A in the
load.
3-29) α ≈ 60.5°. Do a parametric sweep for alpha. Use the default (Dbreak) diode, and
use Ron = 0.01 for the switch. Alpha of 60.5 degrees results in approximately 1.8 A in
the load.
3-30) From Eq. 3-61,

a) i (t ) = 4.29sin(t − 1.263) − 4.0 + 7.43e −t /3.142 A., 0.873  t  3.95 rad

1
2 
Io = i (t )d (t ) = 1.04 A., Pdc = I oVdc = (1.04)(48) = 50.1 W .


1 2
b) I rms =  i (t )d (t ) = 1.67 A.; PR = I rms R = (1.67) 212 = 33.5 W .
2

2 
P 50.1 + 33.5
c) pf = = = 0.417 = 41.7%
S (120)(1.67)
3-31) From Eq. 3-61,

a) i (t ) = 2.95sin(t − 0.515) − 0.96 + 3.44e −t /0.565 A., 1.047  t  3.32 rad

1
2 
Io = i (t )d (t ) = 0.454 A., Pdc = I oVdc = (0.454)(96) = 43.6 W .


1 2
b) I rms =  i (t )d (t ) = 0.830 A.; PR = I rms R = (0.830) 2100 = 69.0 W .
2

2 
P 43.6 + 69.0
c) pf = = = 0.565 = 56.5%
S (240)(0.830)
3-32) α ≈ 75°. Alpha = 75 degrees gives 35 W in the dc voltage source. An Ron =
0.01 for the switch and n = 0.001 for the diode (ideal model).
3-33) From Eq. 3-61,

a ) i (t ) = 5.99sin(t − 1.50) − 24.0 + 29.3e −t /14.1 A., 0.873  t  4.24 rad

1
2 
Io = i (t )d (t ) = 1.91 A., Pdc = I oVdc = (1.91)(48) = 91.6 W .


1 2
b) I rms =  i (t )d (t ) = 2.93 A.; PR = I rms R = (2.93) 2 2 = 17.1 W .
2

2 
3-34) α ≈ 81°

3-35)
di (t )
L = Vm sin t − Vdc
dt
di (t ) 1
= [Vm sin t − Vdc ] or
dt L
di (t ) 1
= [Vm sin t − Vdc ]
d (t )  L
t
1
 L  m
i (t ) = (V sin t − Vdc )d (t )

Vm V
= (cos  − cos t ) + dc ( − t )
L L
i (t ) = 4.34 − 7.58cos t − 1.82t A., 1.309  t  4.249

1
2 
Io = i (t )d (t ) = 1.91 A.
3-36)
v0 = vs when S1 on, v0=0 when D2 on

Vo 1 V
Io =
R
, Vo = 
2 
Vm sin(t )d (t ) = m (1 + cos  )
2
Vm
 Io = (1 + cos  )
2 R

3-37)

 I X 
u = cos −1 1 − L s  ; X s =  Ls = 377(1.5)(10) −3 = 0.566 
 Vm 
 5(0.452) 
u = cos −1 1 −  = 10.47
 120 2 
Vm  X L X s  120 2  5(.566) 
Vo = 1 − = 1 −  = 53.57 V .
  2Vm    2 2(120) 
V
(compared to m = 54.0 V .)

PSpice:
Use a current source for the constant load current:
D1 to D2

D2 to D1
3-38) u = 20°. Run the simulation long enough for steady-state results. From the Probe
output, the commutation angle from D1 to D2 is about 20 degrees, and from D2
to D1 is about 18 degrees. Note that the time axis is changed to angle in degrees
here.
3-39) Run the simulation long enough for steady-state results. From the Probe output, the
commutation angle from D1 to D2 is about 16.5 degrees, and from D2 to D1 is
about 14.7 degrees. Note that the time axis is changed to angle in degrees here.
3-40) At ωt = π, D2 turns on, D1 is on because of the current in LS (see Fig. 3-17).
diD1 did 1
KVL; vLS = Vm sin t = Ls =  LS
dt d (t )
t
diD1 V
= m
d (t )  Ls  sin(t )d (t ) + i D1 ( )

Vm
at t =  + u , iD1 = 0 = [−1 − cos( + u )] + I L
 Ls
Vm V
cos( + u ) = − cos u → 0 = (−1 + cos u ) + I L = − m cos u + I L
 Ls  Ls
 I X 
 u = cos −1 1 − L s 
 Vm 

3-41) At ωt = α,
t
1 V
is (t ) = 
 Ls 
Vm sin(t )d (t ) + 0 = m [cos  − cos t ]
 Ls
Vm
iD 2 ( wt ) = I L − is = I L − [cos  − cos t ]
 Ls
Vm
iD 2 ( + u ) = 0 = I L − [cos  − cos( + u )]
 Ls
I L Ls
− = − cos  + cos( + u )
Vm
 I X 
u = cos −1 cos  − L s  − 
 Vm 
3-42) A good solution is to use a controlled half-wave rectifier with an inductor in series
with the 48-V source and resistance (Fig. 3-15). The switch will change the delay
angle of the SCR to produce the two required power levels. The values of the
delay angle depend on the value selected for the inductor. This solution avoids
adding resistance, thereby avoiding introducing power losses.
3-43) Several circuit can accomplish this objective, including the half-wave rectifier of
Fig. 3-2a and half-wave rectifier with a freewheeling diode of Fig. 3-7, each with
resistance added. Another solution is to use the controlled half-wave rectifier of
Fig. 3-14a but with no resistance. The analysis of that circuit is like that of Fig. 3-
6 but without Vdc. The resulting value of α is 75°, obtain from a PSpice simulation.
That solution is good because no resistance is needed, and losses are not
introduced.
3-44 and 3-45) The controlled half-wave rectifier of Fig. 3-15 (without the resistance) can
be used to satisfy the design specification. The value of the delay angle depends
on the value selected for the inductor.

CHAPTER 4 SOLUTIONS
2/17/10

4-1) Load:

2Vm Vo 2 2(120) / 
Vo = = ; Io = = 6.0 A.
 R 18
V 120 2 9.43
I o , peak = m = = 9.43 A.; I o ,rms = = 6.67 A.
R 18 2

Each diode:

I o 6.0 I
I D ,avg = = = 3.0 A.; I D , peak = I o, peak = 9.43 A.; I D, rms = o ,rms = 4.71 A.
2 2 2
4-2)

2Vm 2 (120 ) 2 Vo 108


bridge : Vo = = = 108 V .; I o =
= = 4.32 A.
  R 25
PIV = Vm = 120 2 = 170 V .
Center tapped : Vm = 120 2, I o = 4.32 A.; PIV = 2Vm = 2(120) 2 = 340 V .

4-3)
V0 2Vm 200
= I0 =
= = 4.24 A.
R  R  15
2V  1 1  Vn
Vn = m  −  ; Z n = R + (no L) ; I n =
2 2

  n −1 n +1  Zn
V2 = 42.4, V4 = 8.49, ; Z 2 = 47.7 , Z 4 = 91.7 
42.4 V
I2 = = 0.890 A., I 4 = 4 = 0.0925 A.
47.7 Z4
2
 0.890  (0.0925) 2
I rms =  I = 4.24 + 
2
n
2
 + = 4.29 A.
 2  2
1 I
I D = I 0 = 2.12 A.; I D ,rms = o ,rms = 3.03 A.
2 2
I s = 0; I s ,rms = I o ,rms = 4.29 A.

4-4)
V0 2Vm 340
I0 == = = 10.8 A.
R  R  10
2V  1 1  Vn
Vn = m  −  ; Z n = R + (no L) ; I n =
2 2

  n −1 n +1  Zn
V2 = 72.2, V4 = 14.4, ; Z 2 = 21.3 , Z 4 = 39.0 
72.2 V
I2 = = 3.38 A., I 4 = 4 = 0.37 A.
21.3 Z4
2
 3.38  (0.37) 2
I rms =  I = 10.8 + 
2
n
2
 + = 11.1 A.
 2  2
1 I
I D = I 0 = 5.4 A.; I D ,rms = o ,rms = 7.84 A.
2 2
I s = 0; I s ,rms = I o ,rms = 11.1 A.

4-5)
a) Average load current : R = 15 L = 30 mH
Vo 2Vm /  2(120) 2 /  108
Io = = = = = 7.20 A.
R R 15 15

b) Power is determined from the Fourier series. Using Eq. 4-4


and 4-5.
n Vn, V. Zn. Ω In, A.
2 72.0 27.1 2.65
4 14.4 47.7 0.302

2 2
 2.65   0.32 
I rms  7.202 +   +  = 7.45 A.; P = I rms R = (7.45) 25 = 832 W .
2 2

 2   2 
P 832
I s ,rms = I o ,rms = 7.45 A.; pf = = = 0.93
S (120)(7.45)

4-6

a) Average load current : R = 12 L = 20 mH


Vo 2Vm /  2(120) 2 /  108
Io = = = = = 9.0 A.
R R 12 12
b) Power is determined from the Fourier series. Using Eq. 4-4
and 4-5.
n Vn, V. Zn. Ω In, A.
2 72.0 19.3 3.74
4 14.4 32.5 0.444

2 2
 3.74   0.444 
I rms  9.02 +   +  = 9.39 A.; P = I rms R = (9.39) 12 = 1, 058 W .
2 2

 2   2 
P 1058
I s ,rms = I o ,rms = 9.39 A.; pf = = = 0.94
S (120)(9.39)

4-7)

Vo 2Vm 2 2(40)
a) I o = = = = 9.0 A.
R R 4
I V 40
b) I rms = m = o ,rms = = 10 A.
2 R 4
c) I s ,avg = 0
N   40 
d ) I s ,rms = I o ,rms  2  = 10   = 1.67 A.
 N1   240 
4-8) Load:
2Vm
Vo = I o R = (10)(15) = 150 V =

Vo 150
Vm = = = 2.36 V .
2 2
V 236
Vo ,rms = m = = 166.6 V . on each sec. tap, 333.2 V . overall
2 2
N V 120
120 V . source : 1 = 1 = = 0.36:1 or 1:2.78
N 2 V2 333.2
N1 240
240 V . source : = = 0.72:1 or 1:1.39
N 2 333.2
4-9)
2Vm
V = I o R = (5)(10) = 50 V . =

Vo 50
Vm = = = 78.5 V .
2 2
V 78.5
Vo ,rms = m = = 55.5 V . on each sec. tap, 111 V . overall
2 2
N V 120
120 V . source : 1 = 1 = = 1.08:1
N 2 V2 111
N1 240
240 V . source : = = 2.16:1
N 2 111

4-10)
Vo 2Vm
a) I o = 10 A. =
=
R R
2V 2(120) 2
R= m = = 10.8  total
 Io  10
Rx = 10.8 − 4 = 6.8 
b) V2 = 72 Z 2 = 151
V2 72
I2 = = = 0.4764
Z 2 151
I o  2 I 2 = 2(0.4764) = 0.953 A.
4-11)

2Vm 2(120) 2
− Vdc − 48
V −V
a) I o = o dc =  =  = 20.0 A.
R R 3
Pdc = I oVdc = (20.0)(48) = 961 W .

b) Fourier Series
n Vn, V. Zn. Ω In, A.
2 72.2 11.7 6.16
4 14.4 22.8 0.631

2 2
 6.16   0.631 
I rms = 20.02 +   +  = 20.5 A.
 2   2 
PR = I rms
2
R = (20.5) 2 (3) = 1, 259 W .
 170  P 961 + 1259
c) S = Vrms I rms =   (20.5) = 2460 VA; pf = S = 2460 = .90
 2
d ) I o = 2 I 2 = 6.16 = 12.32 A.

_____________________________________________________________________________
________

4-12
2Vm 2(340)
− Vdc − 96
V −V
a) I o = o dc =  =  = 24.1 A.
R R 5
Pdc = I oVdc = (24.1)(96) = 2,313 W .

b) Fourier Series
n Vn, V. Zn. Ω In, A.
2 144.3 30.6 4.72
4 28.9 60.5 0.477
2 2
 4.72   0.477 
I rms = 24.1 + 
2
 +  = 24.3 A.
 2   2 
PR = I rms
2
R = (24.1) 2 (5) = 1,958 W .
 340  P 2313 + 1958
c) S = Vrms I rms =   (24.3) = 5,848 VA; pf = S = = .90
 2 5848
d ) I o = 2 I 2 = 4.72 = 9.44 A.

4-13) I 0 = 7.03 A. from PSpice

4-14) a) Continuous current; P=474 W.


b) Discontinuous current; P=805 W.

4-15

2Vm 2(120 2)
− Vdc − 24
Vo − Vdc
a) I o = =  =  = 21.0 A.
R R 4
Pdc = I oVdc = (21.0)(24) = 504 W .

b) Fourier Series
n Vn, V. Zn. Ω In, A.
2 72.0 30.4 2.37
4 14.4 60.5 0.238

2 2
 2.37   0.238 
I rms = 21 + 
2
 +  = 21.1 A.
 2   2 
PR = I rms
2
R = (21.1) 2 (4) = 1, 777 W .
P 504 + 1777
c) S = Vrms I rms = (120 ) (21.1) = 2,529 VA; pf = = = .90
S 2529
4-16

2Vm 2(120 2)
− Vdc − 36
Vo − Vdc
a) I o = =  =  = 14.4 A.
R R 5
Pdc = I oVdc = (14.4)(36) = 519 W .

b) Fourier Series
n Vn, V. Zn. Ω In, A.
2 72.0 45.5 1.58
4 14.4 90.6 0.159

2 2
 1.58   0.159 
I rms = 14.4 + 2
 +  = 14.45 A.
 2   2 
PR = I rms
2
R = (14.45) 2 (5) = 1, 044 W .
P 519 + 1044
c) S = Vrms I rms = (120 ) (14.45) = 1, 734 VA; pf = = = .90
S 1734
_____________________________________________________________________________
________

4-17)
26.5A

100uH

20.0A

40mH

10.0A

0A
150ms 152ms 154ms 156ms 158ms 160ms 162ms 164ms 166ms 168ms 170ms 172ms 174ms
I(L1)
Time

The current with the 100 μH inductor is discontinuous.

4-18)
Vm
V  ; Vo  Vm 120 2 = 169.7 V .; 0.01Vo  1.7 V .
2 fRC
Vm 169.7
C= = = 4160  F .
2 fRVo 2(60)(200)(1.7)
I o Vo 169.7
ID = =  = 0.43 A.
2 2 R 2(200)
I D , peak : from Eq. 4 − 11,
 Vo  −1  1.7 
 = sin −1 1 −  = sin 1 −  = 81.9
 Vm   169.7 
 sin  
From Eq. 3 − 48, I D , peak = Vm  C cos  + 
 R 
 sin 81.9 
= 120 2  377(8.32)(10) −3 cos81.9 +  = 38.5 A.
 200 
4-19)
Vo 100
Req = = = 200 ; Vm = 100  Vo ; Vo = 1 V .
I o 0.5
Vm 100
C= = = 4167  F .
2 fRC Vo 2(60)(200)(1)
Io
ID = = 0.2 A.
2
 V 
From Eq. 4 − 11,  = sin −1 1 − o  = sin −1 (0.99) = 81.9
 Vo 
 sin  
From Eq. 3 − 48, I D , peak = Vm   C cos  + 
 R 
 sin 81.9 
= 100  377(4167)(10) −6 cos81.9 +  = 22.6 A.
 200 
4-20) C ≈ 3333/2 = 1667 µF. Peak diode currents are the same. Fullwave circuit has
advantages of zero average source current, smaller capacitor, and average diode current
½ that for the halfwave. The halfwave circuit has fewer diodes, and has only one diode
voltage drop rather than two.

4-21)
3 L 3(377)(.01)
a) R = 7  : = = 1.62
R 7
> 1  continuous current
2Vm
Vo = = 108 V .

3 L 3(377)(.01)
b) R = 20  : = = 0.57
R 20
From Fig . 4 − 8, Vo  0.7Vm = 0.7(120) 2 = 119 V .
V   119 
(1) Eq. 4 − 18 :  = sin −1  o  = sin −1   = 0.777 rad .
 Vm   120 2 
1
(2) Eq. 4 − 20 : I L (t ) = [V (cos  − cos t ) − Vo (t −  )] < 1 
L m
iL (  ) = 0 = Vm (cos  − cos  ) − Vo (  −  )
= 120 2(cos(.777) − cos  ) − 119(  − .777) →  = 3.216 rad .

1
(3) I L =
  i (t )d (t ) = 6.14 A.

L

(4) Vo = I L R = (6.14)(20) = 122.9 V .  119 V .


Calculated Vo is slightly larger than initial estimate. Try Vo=120 V.:
(1) Vo = 120 V . From Eq. 4 − 18,  = 0.785
(2) From Eq. 4 − 20,
i (  ) = 0 = 120 2[cos(.785) − cos  ] − 120(  − .785) →  = 3.197 rad .

1
(3) I L =
 
 i(t )d (t ) = 5.895 A.
(4) Vo = I L R = (5.895)(20) = 117.9 V .  120

Therefore, 119 < Vo < 120 V. (Vo=119.6 with more iterations.)

c) PSpice results:

127

R=20

R=7
100

50

R=7

R=20
0
559ms 560ms 561ms 562ms 563ms 564ms 565ms 566ms 567ms 568ms 569ms 570ms
V(OUT+,OUT-) I(L1)
Time

R = 7 results in continuous current with Vo = 108 V. R = 20 results in discontinuous current


with Vo = 120 V. The simulation was done with C = 10,000 μF.
4-22) PSpice results with a 0.5 Ω resistance in series with the inductance: For Rload =
5 Ω, Vo=56.6 V. (compared to 63.7 volts with an ideal inductor); for Rload = 50 Ω,
Vo=82.7 V. (compared to 84.1 volts with an ideal inductor).
4-23)

Vm 120 2
a) I o = (1 + cos  ) = (1 + cos 45) = 4.61 A.
R  20
V 1  sin  120 2 1 45( /180) sin 90
b) I rms = m − + = − + = 5.72 A.
R 2 2 4 20 2 2 4
c) I s ,rms = I o ,rms = 5.72 A.
d ) P = I rms
2
R = (5.72) 2 20 = 655 W .; S = Vrms I rms = (120)(5.72) = 686 VA.
P 655
pf = = = 0.954
S 686
4-24)

1  sin 2
Vm
I rms = − +
2 2
R 4
V 
S = Vrms I rms =  m  ; P = I rms
2
R
 2
Vm 1  sin 2
2 − +
2 2 4
2
P I rms R 2 I rms R R
pf = = = =
S  Vm  Vm Vm
  I rms
 2
1  sin 2  sin 2
= 2 − + = 1− +
2 2 4  2
4-25) a) α = 15° : Check for continuous current. First period:
Vm
i (t ) = sin((t ) −  ) + Ae −t / = 10.84sin(t − 0.646) + 5.75e −t /.754
Z
i (  ) = 0 →  = 217;  − 180 = 37   → continuous current
Or
L 377(0.050)
 = tan −1 = tan −1 = 37   → continuous current
R 25
2V V 208.7
Vo = m cos  = 208.7 V .; I o = o = = 8.35 A.
 R 25

b) α = 75° Check for continuous current. First period:


 = 37 from part a,  = 75→ discontinuous current
V
i (t ) = m sin((t ) −  ) + Ae −t / = 10.84sin(t − 0.646) − 37.9e −t /0.754
Z
i (  ) = 0 →  = 216 ;  − 180 = 36   → discontinuous current

1
Io =
  i(t )d (t ) = 2.32 A.

4-26)a) α = 20°: Check for continuous current. First period:
Vm
i (t ) = sin((t ) −  ) + Ae −t / = 4.12sin(t − 0.756) + 2.36e −t /0.943
Z
i (  ) = 0 →  = 224,  − 180 = 42   → continuous current
L 377(0.075)
 = tan −1 = tan −1 = 43   → continuous current
R 30
2V V 101.5
Vo = m cos  = 101.5 V .; I o = o = = 3.38 A.
 R 30

b) α = 80°: Check for continuous current. First period:


V
i (t ) = m sin((t ) −  ) + Ae −t / = 4.12sin(t − 0.756) − 10.8e −t /0.943
Z
i (  ) = 0 →  = 221;  − 180 = 41   → discontinuous current
L
 = tan −1 = 37   → discontinuous current
R

1
Io =
  i(t )d (t ) = 0.838 A.

4-27) The source current is a square wave of ±Io.


V 2V
P = I rms
2
R = I o2 R; I o = o = m
pf R R
2 2
 2V   2V  1
P= m  R= m 
 R     R
Vm  2Vm  2Vm2
S = Vs ,rms I s ,rms = Vs ,rms I o =  =
2  R  R
2
 2Vm  1
P    R 2 2
pf = =  =
S 2Vm2 
R
4-28)

I o = 4.5 A.  Vo = I o R = 4.5(20) = 90 V .

I o = 8 A.  Vo = I o R = 8(20) = 160 V .
Vm 160
Eq. 4 − 23 : Vo = (1 + cos  ): forVo = 160 V . and  = 0, Vm = =2
 2
V    90 
forVm = 251 and Vo = 90,  = cos −1  o − 1 = cos −1  − 1 = 82.7
 Vm   251 
Vm' 120 2
turns ratio = = = 0.68 :1 or 1:148
Vm 251
Note that the turns ratio could be lower (higher secondary voltage) and α adjusted
accordingly.

4-29)

Vo = I o R = 10(5) = 50 V .; from Eq. 4 − 30,


 Vo   50 
 = cos −1  −1
 = cos   = 62.5

 2Vm   2 2 (120 ) 
 L  −1  377(.1) 
check for continuous current : tan −1   = tan   = 82.4
 R   5 
62.5  82.4  continuous
V2 = 132 V . Z 2 = 75.6  I 2 = 1.75 A.
I o  2(1.75) = 3.5 A.
4-30)

2Vm 2 2(240)
Vo = cos  = cos105 = −56 V .
 
100 − 56
Io = = 4.4 A.; Pdc = I oVdc = (4.4)(100) = 440 W .
10
Pac = Pbridge = I oVo = (4.4)(56) = 246 W .
PR = Pdc − Pac = 440 − 246 = 194 W .
V2
From Fig. 4 − 12,  0.83 for  = 105
Vm
V2 = 0.83 Vm = 0.83 2(240) = 281 V .
Z 2 = R + j 2 L = 10 + j 2(377)(.8) = 603 − R
V2 281
I2 = = = 0.47 A.; I o  2 I 2 = 0.94 A. p − p
Z 2 603
4-31)
Vo − Vdc
a) I o =
R
 V −V  1 2
Pbridge (absorbed )  I o (−Vo ) =  o dc  (−Vo ) = −   (Vo − VoVdc )
 R  R
Vo2 − VoVdc + Pbr R = 0
Vo2 + 100Vo + 2000(0.8) = 0
Vo = −20 V or − 80 V .
2000 2000
with Vo = −20, I o = = 100 A.; with Vo = −80, I o = = 25 A.
20 80
choose Vo = −80 V . to minimize losses
 Vo  −1  −80 
 =cos −1   = cos  137.8
 2Vm   2 2(120 
V2
b) at  = 137.8, from Fig. 4 − 12,  0.65  V2 = 0.65 2(120) = 110 V .
Vm
I o  (.1) I o = (.1)(25) = 2.5 A.
I o
I2 = = 1.25 A.
2
V 110
Z2 = 2 = = 88  = R + j 2o L  2o L
I 2 1.25
Z2 88
L= = = 0.117 H = 117 mH
2o 2(377)
Choose L somewhat larger, say 120 mH, to allow for approximations.
4-32) In Fig. 4-14, Pac = Pbridge = -VoIo = 1000 W. Using Vdc = -96 V gives this
solution:
Kirchhoff ' s voltage law gives − Vo + (1) I o − 96 = 0

−1000
Vo =
Io

1000
+ I o − 96 = 0
Io

I o2 − 96 I o + 1000 = 0

I o = 84.11 or 11.89 A. Use11.89 A.

then Vo = −84.11V .

 Vo   −84.11 
 = cos −1  −1
 = cos   = 141.1
 2Vm   2 (120 ) 2 

From Fig .4 − 14,
V2
Vm
(
 0.64 gives V2 = 0.64 120 2 = 109 V . )
1.189
I o  2 I 2 = 0.10 ( I o ) = 1.189 A. I2 = = 0.595 A
2
V2 109
Z2 = = = 183 = R + j L = 1 + j L
I 2 0.595

183 183
 L  183 L = = 0.48 H .
 377

_____________________________________________________________________________
________
4-33)
a ) Pdc = 5000 W . absorbed → I oVdc = −5000
−5000
Io = = 33.3 A.
−150
Vo = −150 + 0.6 I o = −150 + 0.6(33.3) = −130 V .
2Vm V    −130 
Vo = cos  →  = cos −1  o  = cos −1   = 127
  2Vm   2 2(240) 
b) Pbridge = I o (−Vo ) = (33.3)(130) = 4329 W .
V2
c) From Fig . 4 − 12, at 127,  0.73 → V2 = 0.73(240) 2 = 248 V .
Vm
I o
I o = 0.1I o = 0.1(33.3) A.; I 2 = = 1.67 A.
2
V2 248
Z2 = = = 149   2o L
I 2 1.67
149
L= = 0.197 H  200 mH
2(377)
4-34)

3Vm 3 2(480)
a ) Vo = = = 648 V .
 
Vo 648
Io = = = 12.96 A.
R 50
V 480 2  2
b) io (t ) = m sin t = sin t = 13.6sin t for  t 
R 50 3 3
2 /3
1
I rms =
 /3 
 /3
(13.6sin t ) 2 d (t ) = 12.98 A.

2
I s ,rms = (12.98) = 10.6 A.
3
c) P = I rms
2
R = (12.98) 2 50 = 8419 W .
S = 3VI = 3(480)(10.6) = 8808 VA
P 8419
pf = = = 0.956
S 8808

4-35)
3Vm 3 2(240) Vo 324
a) Vo = = = 324 V .; I o = = = 4.05 A.
  R 80
6Vm
b) V6 = = 0.055Vm = 0.055 2(240) = 18.5 V .
 (62 − 1)
Z 6 = R = 80
V6 18.5
I6 = = = 0.23 A.
Z6 80
2
 0.23 
I rms  I + I 6 rms = 4.05 + 
2
o
2
 = 4.06 A.
 2 
I 4.04
c) I D = o = = 2.02 A.
2 2
I 4.05
d ) I D ,rms = o ,rms = = 2.87 A.
2 2
I o ,rms 2 4.06 2
e) I s ,rms = = = 3.31 A.
3 3
f) P=I 2
o , rms R = (4.06) 2 80 = 1315 W .; S = 3VI = 3(240)(3.31) = 1376 VA
P 1315
pf = = = 0.956
S 1376
4-36)

3Vm 3 2(480) Vo 649


a) Vo = = = 649 V .; I o = = = 6.49 A.
  R 100
6Vm
b) V6 = = 0.055Vm = 0.055 2(480) = 37.1 V .
 (62 − 1)
Z 6 = R + j 6o L = 100 + j 6(377)(.015) = 100 + j 37.9 = 106 
V6 37.1
I6 = = = 0.35 A.
Z 6 106
2
 0.35 
I rms  I + I 6 rms = 6.49 + 
2
o
2
 = 6.49 A.
 2 
I 6.49
c) I D = o = = 3.25 A.
2 2
I 6.49
d ) I D ,rms = o ,rms = = 4.59 A.
2 2
I o ,rms 2 6.49 2
e) I s ,rms = = = 5.3 A.
3 3
f) P=I 2
o , rms R = (6.49) 2100 = 4212 W .; S = 3VI = 3(480)(5.3) = 4406 VA
P 4212
pf = = = 0.956
S 4406
4-37)
There are no differences between the calculations in Problem 4.36 and the PSpice
results. The power absorbed by each diode ia approximately 1.9 W.
4-38)Equation (4-46) gives values of of I1 = 28.6 A, I5 = 5.71 A, I7 = 4.08 A, I11 = 2.60
A, and I13 = 2.20 A. All compare well with the PSpice results. The total harmonic
distortion (THD) is 27.2% when including harmonics through n = 13.

_____________________________________________________________________________
________

4-39)
a ) Vo = I o R = (25)(120) = 3000 V .
  Vo  −1   3000 
 = cos −1   = cos   = 57.7
 3Vm   3 2(4160) 
V6
b) From Fig. 4 − 21,  0.28  V6 = 0.28 2(4160) = 1640 V .
Vm
V12
 0.135  V12 = 794 V .
Vm
V18
 0.09  V18 = 525 V .
Vm

c)

50A

0A Load

-50A
I(R)

40A
S1

0A
I(S1)
80A

S4
SEL>>
0A
I(S4)
50A
Ia
0A

-50A
65ms 70ms 75ms 80ms 85ms 90ms 95ms 100ms
-I(VAN)
Time

4-40)
a ) Vo = I o R = (10)(50) = 500 V .
  Vo  −1   500 
 = cos −1   = cos   = 39.5
 3Vm   3 2(480) 
V6
b) From Fig. 4 − 21,  0.21  V6 = 0.21 2(480) = 143 V .
Vm
V12
 0.1  V12 = 68 V .
Vm
V18
 0.07  V18 = 48 V .
Vm

c)

20A

SEL>> Load
-20A
I(R)

S1
10A

0A
I(S1)

10A S4

0A
I(S4)

10A
0A Ia

-10A

65ms 70ms 75ms 80ms 85ms 90ms 95ms 100ms


-I(VAN)
Time

_____________________________________________________________________________
________
4-41)

3Vm 3 2(480)
a) Vo = cos  = cos 35 = 531 V .
 
Vo 531
Io = = = 10.6 A.
R 50
V
b) 6  0.19  V6 = 0.19 2(480) = 130 V .
Vm
Z 6 = R + j 60 L = 50 + j 6(377)(0.05) = 124 
V6 130
I6 = = = 1.05 A.
Z 6 124
2 2
 I   1.05 
I o ,rms  i +  6  = 10.62 + 
2
o  = 10.65 A.
 2  2 
 2  2
I s ,rms =   I o ,rms =  10.65 = 8.6 A.
 3  3
4-42)

3Vm 3 2(480)
a ) Vo = cos  = cos 50 = 417 V .
 
Vo 417
Io = = = 41.7 A.
R 10
V
b) 6  0.25  V6 = 0.25 2(480) = 170 V .
Vm
Z 6 = R + j 60 L = 10 + j 6(377)(0.01) = 24.7 
V6 170
I6 = = = 6.9 A.
Z 6 24.7
2 2
 I   6.9 
I o ,rms  i +  6  = 41.7 2 + 
2
o  = 42.3 A.
 2  2
 2  2
I s ,rms =   I o ,rms =   41.7 = 34 A.
 3   3 
4-43)
a ) Vo = I o R = (20)(20) = 400 V .

 V    400 
a = cos −1  o  = cos −1   = 52
 3Vm   3 2(480) 
V
b) From Fig . 4 − 21, 6  0.25  V6 = 0.25( 2)(480) = 170 V .
Vm
2 2 2
 I 6   I12   I18 
  +  +   0.02 I o or I 62 + I122 + I182  0.02 2 I o
 2  2  2
Z 6 = R + j 6 L
V6
= I 6  0.02 I o = 0.02(20) = 0.4 A.
Z6
V6 170
Z6 = = = 425  = R + j 6 L = 20 + j 6(377) L
I 6 0.4
6(377) L  425
425
L= = 0.188 H
6(377)
L  190 mH
4-44)

 Vo   −280 
 = cos −1  −1
 = cos   = 149.8
 3 2 ( 280 ) 
 3Vm  

300V − 280V
Io = = 40 A
0.5
Pdc = Vdc I o = ( 300 )( 40 ) = 12,000 W supplied

PR = I o2 R = 402 ( 0.5 ) = 800 W absorbed

PBridge = Pac = ( 280 )( 40 ) = 11, 200 W absorbed

_____________________________________________________________________________
________
4-45)

 1.5(10)6 W . 
Pbridge = 1.5 MW .; Vo =   = −1500 V .
 1000 A. 
 V    (−1500) 
 = cos −1  o  = cos −1   = 105.5
 3Vm   3 2(4160) 
 2
I s ,rms =  1000 = 816 A.
 3 

4-46)
−100,000
With Pac = Pbridge = 100 kW absorbed, - Vo I o = 100,000 or Vo =
Io

Kirchhoff's voltage law gives - Vo + ( I o ) ( 0.1 ) − 1000V = 0

−100,000
Substituting for Vo , − + 0.1I o − 1000 = 0
Io

0.1I o2 − 1000 I o + 100,000 = 0 yields the two roots Io = 9,890 A or 101 A

Choose Io = 101 A because this solution results in lower I o2 losses.

Vo = −1000V + I o ( 0.1 ) = −1000 + ( 0.1)101 = −989.9 V

3Vm
Vo = cos  , where Vm = 2 12,500 ( N 2 / N1 ) 

 Vo   −989.9 
 = cos −1  −1
 = cos  
 3Vm  
 3 2 
12,5 00 ( N 2 / N ) 
1 

N 2 / N1 = 1 will theoretically work, but  = 93.36, but the harmonic content will be large.

A better solution would be to choose N 2 / N1 to be perhaps 1/10 (step-down). Then  = 125.9


V6
From Fig. 4-21,  0.3
Vm

Vm = 2 (12,500 ( N 2 / N1 ) ) = 2 (12,500 / 10 ) = 1768 V

V6  0.3Vm = 0.3 (1768 ) = 530V

I o  2 I 6  0.5I o = 0.5 (101) = 5.05 A → I 6 = 2.525 A

V6 530
Z6 = = = 210  = R + j L = 0.1 + j 377 L  377 L
I 6 2.525

210
L=
= 0.56 H
377
_____________________________________________________________________________
________
4-47)

a) Vo1 =
3Vm, L − L
cos(1 ) =
(
3 230 2 ) cos(45) = 329.5 kV
 

Vo 2 =
3Vm, L − L
cos( 2 ) =
(
3 230 2 ) cos(134.4) = −326 kV
 
Vo1 + Vo 2 329.5kV − 326kV
Io = = = 231 A
R 15
P1 = Vo1 I o = 76.17 MW

P2 = Vo 2 I o = −75.37 MW

b) Pline = I o2 R = 800 kW
_____________________________________________________________________________
________

4-48)
3Vm
a ) Vm = 230 2 kV ; Vo = cos( )

Vo,max =
3Vm
=
(
3 230 2 ) = 325.3 kV
 
V  
Let Vo 2 = −300 kV (arbitrarily ); Then  2 = cos −1  o 2  = 164.98
 3Vm 
P2 −80 MW
Io = = = 267 A (linecurrent )
Vo 2 −300kW

Vo1 = I o R − Vo 2 = 267(12) − (−300kV ) = 303.2 kV

Pline = I o2 R = 853 kW
_____________________________________________________________________________
________

4-49)
3Vm
a) Vm = 345 2 kV ; Vo = cos( )

Vo,max =
3Vm
=
(
3 345 2 ) = 465.9 kV
 
V  
Let Vo 2 = −425kV (arbitrarily ); Then  2 = cos −1  o 2  = 155.8
 3Vm 
P2 −300MW
Io = = = 706 A (linecurrent )
Vo 2 −425kW

Vo1 = I o R − Vo 2 = 706(20) − (−425kV ) = 439.1kV

Pline = I o2 R = 9.97 MW

_____________________________________________________________________________
________
4-50)
8 A.  I o  12 A.
 2V 
Vo =  m  cos  = I o R; Vo1 = 8(8) = 64 V .; Vo 2 = 12(8) = 96 V .
  
 64 
120 − volt source : 1 = cos −1   = 53.7
 2 2(120) 
V
From Fig . 4 − 12, 2  0.73  V2 = 124 V .
Vm
 96 
 2 = cos −1   = 27.3
 2 2(120) 
V
From Fig . 4 − 12, 2  0.54  V2 = 92 V .
Vm
V2 124
using V2 = 124 V . for 1 and I o  2.5 A., Z 2 = = = 99 
I 2 2.5 / 2
Z2 99
Z 2 = R + j 20 L  L = = = 0.13 H
20 2(377)
For the 240 − volt source,
 64 
1 = cos −1   = 72.8
 2 2(240) 
V
From Fig . 4 − 12, 2  0.83  V2 = 280 V .
Vm
 96 
 2 = cos −1   = 63.6
 2 2 ( 240 ) 
 
V
From Fig . 4 − 12, 2  0.78  V2 = 265 V .
Vm
V2 280
using V2 = 280 V . for 1 and I o  2.5 A., Z 2 = = = 224 
I 2 2.5 / 2
Z2 244
Z 2 = R + j 20 L  20 L  L = = = 0.3 H
20 2(377)
The 120-volt source requires a smaller filter inductor.
_____________________________________________________________________________
________

4-51)
Io =15A in a 20- resistor. Io = 0.1(Io ) = 1.5 A.

First solution using the single-phase 480-V source with a controlled bridge rectifier:

 Vo 
 = cos −1   = 46
 2Vm 
I o  2 I 2 → I 2 = 1.5 / 2 = 0.75 A

From Fig. 4-12, V2 /Vm  0.71 → V2 = 0.71 2 ( 480 ) = 482V

V2 482
Z2 = = = 643  = R + jn L = 20 + j 2 ( 377 ) L
I 2 0.75

642
2 ( 377 ) L  643 → L = = 851 mH
2 ( 377 )

Second solution using the three-phase 480-V source with a controlled 6-pulse bridge rectifier:

 Vo   300 
 = cos −1  −1
 = cos   = 62.4
 3V  3 2 ( 480 ) 
 m, L − L   
I o  2 I 6 → I 6 = 1.5 / 2 = 0.75 A

From Fig. 4-12, V6 /Vm  0.28 → V6 = 0.28 2 ( 480 ) = 190V

V6 190
Z6 = = = 253  = R + jn L = 20 + j ( 6 ) 377 L
I 6 0.75

253
6 ( 377 ) L  253 → L = = 112 mH
6 ( 377 )

Uncontrolled rectifiers with additional resistances added can also satisfy the
specifications. However, adding resistance would increase power loss and decrease
efficiency.
_____________________________________________________________________________
________

CHAPTER 5 SOLUTIONS

3/9/10
5-1)

Vm  sin(2 )  sin(2 )
a) Vo,rms = 1− + = Vrms 1 − +
2  2  2
Vo,rms 431
Vm = 480 2  = 60  Vo,rms = 431 V ; I o,rms = = = 8.61 A
R 50
Vo2,rms 4312
b) P= = = 3708 W
R 50
P P 3708
c) pf = = = = 0.897
S Vrms I rms ( 480 )( 8.61)

Vm 480 2
d ) I avg , SCR = (1 + cos  ) = (1 + cos 60 ) = 3.24 A
2 R 2 ( 50 )

I o,rms 8.61
I rms , SCR = = = 6.1 A
2 2

 480 
e) I1,rms  0.84   = 8.0 A
 50 
2
I rms − I1,2rms 8.612 − 8.02
THDI = = = 0.38 = 38%
I1,rms 8.0

_____________________________________________________________________________
________
5-2)

Vm  sin(2 )  sin(2 )
a) Vo,rms = 1− + = Vrms 1 − +
2  2  2
Vo,rms 114.4
Vm = 120 2  = 45  Vo,rms = 114.4 V ; I o,rms = = = 5.72 A
R 20
Vo2,rms 114.42
b) P= = = 655 W
R 20
P P 655
c) pf = = = = 0.953
S Vrms I rms (120 )( 5.72 )

Vm 120 2
d ) I avg , SCR = (1 + cos  ) = (1 + cos 45 ) = 2.30 A
2 R 2 ( 20 )

I o,rms 5.72
I rms , SCR = = = 4.05 A
2 2

 120 
e) I1,rms  0.92   = 5.53 A
 20 
2
I rms − I1,2rms 5.722 − 5.532
THDI = = = 0.26 = 26%
I1,rms 5.53

_____________________________________________________________________________
________

5-3)

Vo2.rms
P=  Vo.rms = PR = (800 )( 35) = 167.3 V
R
167.3
from Fig . 5.2, = 0.7    92
240
Vo ,rms 167.3
I o ,rms = = = 4.78 A
R 35
I o ,rms 4.78
I SCR ,rms = = = 3.38 A
2 2
P 800
pf = = = 0.70 = 70%
S 120 ( 4.78 )

_____________________________________________________________________________
________
5-4)

120
With the 240-Vsource, Vo,rms = 120V ; = 0.5 ; a  115 from Fig.5.2
240
 sin(2 )
or solving Eq.5-3, 120 − 240 1 − + = 0   = 1.99 rad = 114
 2

240 V source : Vo, peak = 2 ( 240 ) sin (114 ) = 310 V

120 V source : Vo, peak = Vm = 2 (120 ) = 170V

_____________________________________________________________________________
________

5-5)

For P = 200W , Vo,rms = PR = 200 ( 40 ) = 89.4 V

Using Eq. 5-3,

 sin(2 )
89.4 − 120 1 − + = 0   = 1.48 rad = 85
 2
P P 200
pf = = = = 0.75 = 75%.
S Vrms I rms (120 )( 89.4 / 40 )

For P = 400 W , Vo,rms = PR = 400 ( 40 ) = 126 V

Since 126 V > 120 V of the source, 400 W is not possible.

1202
The maximum power available is = 360 W. The pf is 1.0 for 360 W.
40

_____________________________________________________________________________
________
5-6)

Using the circuit of Fig. 5-1a,

For P = 750W , Vo,rms = PR = 750 ( 32 ) = 154.9 V

Using Eq. 5-3,

 sin(2 )
154.9 − 240 1 − + = 0   = 1.703 rad = 97.6
 2

For P = 1500 W , Vo,rms = PR = 1500 ( 32 ) = 219 V

 sin(2 )
219 − 240 1 − + = 0  0.986 rad = 56.5
 2
Io 219 / 32
Maximum SCR currents are for 1500 W: I SCR ,rms = = = 4.84 A
2 2

2 ( 240 )
(1 + cos ( 56.5 ) ) = 2.62 A
Vm
I SRC ,avg = (1 + cos  ) =
2 R 2 ( 32 )

Vmax = 2(240) = 340 V

_____________________________________________________________________________
________

5-7)

Using the circuit of Fig. 5-1a,

For R = 20, Vo ,rms = PR = 1200 ( 20 ) = 154.9 V

Using Eq. 5-3,

 sin(2 )
154.9 − 240 1 − + = 0   = 1.703 rad = 97.6
 2

For R = 40, Vo,rms = PR = 1200 ( 40 ) = 219 V

 sin(2 )
219 − 240 1 − + = 0  0.986 rad = 56.5
 2
Io 154.9 / 20
Maximum SCR currents are for R = 20 : I SCR ,rms = = = 5.48 A
2 2

2 ( 240 )
(1 + cos ( 97.6) ) = 2.34 A
Vm
I SRC ,avg = (1 + cos  ) =
2 R 2 ( 20 )

Vmax = 2(240) = 340 V


_____________________________________________________________________________
________

5-8)

V 2 1202
R= = = 144 
P 100

a) P = 75 W : Vrms = (144 )( 75 ) = 103.9V


From Fig.5-3,   1.16 rad = 66.2

b) P = 25 W : Vrms = (144 )( 25) = 60 V


From Fig.5-3,   1.99 rad = 114

_____________________________________________________________________________
________

5-9) S1 is on from α to π, and D2 is on from π to 2π.

vo (t ) = Vm sin t for   t  2

1  sin ( 2 )
2
1
 (V sin t ) d (t ) = Vm
2
Vo,rms = − +
2 2 4 8
m

Vm  sin ( 2 )
= 1− +
2 2 4
Vm V
0     Vo,rms  m
2 2

_____________________________________________________________________________
________

5-10)
vo (t ) = Vm sin t for 1  t   and for  +  2  t  2

1  
 2
Vo.rms =  ( m
V sin  t ) ( )
2
d  t +  ( m
V sin  t ) ( )
2
d  t
2   
 1  + 2 

1 1 +  2 sin ( 21 ) + sin ( 2 2 )


= Vm − +
2 4 8

Vm 1 +  2 sin ( 21 ) + sin ( 2 2 )


Vo.rms = 1 − +
2 2 4

_____________________________________________________________________________
________

5-11) a) Using Eq. 5-9,


Z = 21.3 ;  = 0.561 rad ;  = 0.628

i (t ) = 7.98sin (t − 0.561) − 19.25e −t /0.628 A

 = 60 = 1.047 rad ,  = 3.696 rad = 212


b) I rms = 4.87 A

4.87
c) I rms , SCR = = 3.44 A
2

R = ( 4.87 ) (18 ) = 427 W


2
d ) P = I rms
2
_____________________________________________________________________________
________

5-12) Using Eq. 5-9,


Z = 26.7 ;  = 0.601 rad ;  = 0.685

i (t ) = 6.36sin (t − 0.601) − 6.10e −t /0.685 A

 = 50 = 0.873 rad ,  = 3.738 rad = 214


b) I rms = 4.18 A

4.18
c) I rms , SCR = = 2.95 A
2

R = ( 4.18 ) ( 22 ) = 384 W
2
d ) P = I rms
2

_____________________________________________________________________________
________

5-13) Using Eq. 5-9,


Z = 15.0 ;  = 0.646 rad ;  = 0.754

i (t ) = 11.3sin (t − 0.646 ) − 158e −t /0.754 A

 = 115 = 2.01 rad ,  = 3.681 rad = 211


I rms = 2.95 A

_____________________________________________________________________________
________

5-14) Using Eq. 5-9,


Z = 14.2 ;  = 0.561 rad ;  = 0.6.28

i (t ) = 11.98sin (t − 0.561) − 54.1e −t /0.628 A

 = 70 = 1.222 rad ,  = 3.691 rad = 212


I rms = 6.69 A

R = ( 6.69 ) (12 ) = 537 W


2
P = I rms
2

PSpice: P = AVG(W(R)) in Probe gives 523 W (read at the end of the trace). The
difference between PSpice and the theoretical output is because of the nonideal SCR
model in PSpice. The PSpice result will be more realistic. The THD is 22.4% from the
PSpice output file using Fourier terms through n = 9.

_____________________________________________________________________________
________

5-15) Use the PSpice circuit of Example 5-3. The .STEP PARAM command is quite useful for
determining α. (a) α ≈ 81° for 400 W. (b) α ≈ 46° for 700 W.

SINGLE-PHASE VOLTAGE CONTROLLER (voltcont.cir)

*** OUTPUT VOLTAGE IS V(3), OUTPUT CURRENT IS I(R) ***

**************** INPUT PARAMETERS *********************

.PARAM VS = 120 ; source rms voltage


.PARAM ALPHA = 81 ; delay angle in degrees

.STEP PARAM ALPHA 10 90 20 ; try several values of alpha. Modify the range for more precision

.PARAM R = 15 ; load resistance

.PARAM L = 15mH ; load inductance

.PARAM F = 60 ; frequency

.PARAM TALPHA = {ALPHA/(360*F)} ; converts angle to time delay

.PARAM PW = {0.5/F} ; pulse width for switch control

***************** CIRCUIT DESCRIPTION *********************

VS 1 0 SIN(0 {VS*SQRT(2)} {F})

S1 1 2 11 0 SMOD

D1 2 3 DMOD ; forward SCR

S2 3 5 0 11 SMOD

D2 5 1 DMOD ; reverse SCR

R 3 4 {R}

L 4 0 {L}

**************** MODELS AND COMMANDS ********************

.MODEL DMOD D(n=0.01)

.MODEL SMOD VSWITCH (RON=.01)

VCONTROL 11 0 PULSE(-10 10 {TALPHA} 0 0 {PW} {1/F}) ;control for both switches

.TRAN .1MS 50MS 0MS 1u UIC ; one period of output

.FOUR 60 I(R) ; Fourier Analysis to get THD

.PROBE

.END

_____________________________________________________________________________
________

5-16) Modify the PSpice circuit file of Example 5-3. Use the .STEP PARAM command (see Prob.
5-15) for determining α. (a) α ≈ 80° for 600 W. (b) α ≈ 57° for 1000 W.

_____________________________________________________________________________
________
5-17) The single-phase voltage controller of Fig. 5-4a is suitable for this application. Equation
(5-9) applies for each half-period of the input sine wave. For 250 W delivered to the load, each
half period must deliver 125 W. Therefore, the rms value of the current in Eq. (5-9) must be
2.28 A, found by using I2R = 125. A closed-form solution is not possible, but trial-and-error
numerical techniques give α ≈ 74°. A similar but perhaps easier method is to use PSpice
simulations using the PSpice A/D circuit file in Example 5-3. Modifying the diode model to
.MODEL DMOD D(n=.01) to represent an ideal diode, and with trial-and-error values of α, gives
α ≈ 74°.

The average and rms currents are determined from a numerical integration of the current
expression from Eq. (5-9) or from a PSpice simulation. ISCR,avg = 1.3 A, ISCR,rms = 2.3 A. The
maximum voltage across the switches is 120√2sin(74°) = 163 V.

5-18) The PSpice circuit file is shown below. The total average load power is three times the
power in one of the phase resistors. Enter 3*AVG(W(RA)) in Probe. The results are (a) 6.45 kW
for 20°, (b) 2.79 kW for 80°, and (c) 433 W for 115°. Note that the .STEP PARAM command can
be used to run the three simulations at once.

THREE-PHASE VOLTAGE CONTROLLER -- R-L LOAD (3phvc.cir)

*SOURCE AND LOAD ARE Y-CONNECTED (UNGROUNDED)

********************** INPUT PARAMETERS ****************************

.PARAM Vs=480 ; rms line-to-line voltage

.PARAM ALPHA=20 ; delay angle in degrees

.STEP PARAM ALPHA LIST 20 80 115

.PARAM R=35 ; load resistance (y-connected)

.PARAM L = 1p ; load inductance

.PARAM F=60 ; source frequency

********************** COMPUTED PARAMETERS **************************

.PARAM Vm={Vs*SQRT(2)/SQRT(3)} ; convert to peak line-neutral volts

.PARAM DLAY={1/(6*F)} ; switching interval is 1/6 period

.PARAM PW={.5/F} TALPHA={ALPHA/(F*360)}

.PARAM TRF=10US ; rise and fall time for pulse switch control

*********************** THREE-PHASE SOURCE **************************

VAN 1 0 SIN(0 {VM} 60)

VBN 2 0 SIN(0 {VM} 60 0 0 -120)

VCN 3 0 SIN(0 {VM} 60 0 0 -240)


***************************** SWITCHES ********************************

S1 1 8 18 0 SMOD ; A-phase

D1 8 4 DMOD

S4 4 9 19 0 SMOD

D4 9 1 DMOD

S3 2 10 20 0 SMOD ; B-phase

D3 10 5 DMOD

S6 5 11 21 0 SMOD

D6 11 2 DMOD

S5 3 12 22 0 SMOD ; C-phase

D5 12 6 DMOD

S2 6 13 23 0 SMOD

D2 13 3 DMOD

***************************** LOAD **********************************

RA 4 4A {R} ; van = v(4,7)

LA 4A 7 {L}

RB 5 5A {R} ; vbn = v(5,7)

LB 5A 7 {L}

RC 6 6A {R} ; vcn = v(6,7)

LC 6A 7 {L}

************************* SWITCH CONTROL *****************************

V1 18 0 PULSE(-10 10 {TALPHA} {TRF} {TRF} {PW} {1/F})

V4 19 0 PULSE(-10 10 {TALPHA+3*DLAY} {TRF} {TRF} {PW} {1/F})

V3 20 0 PULSE(-10 10 {TALPHA+2*DLAY} {TRF} {TRF} {PW} {1/F})

V6 21 0 PULSE(-10 10 {TALPHA+5*DLAY} {TRF} {TRF} {PW} {1/F})

V5 22 0 PULSE(-10 10 {TALPHA+4*DLAY} {TRF} {TRF} {PW} {1/F})

V2 23 0 PULSE(-10 10 {TALPHA+DLAY} {TRF} {TRF} {PW} {1/F})

************************ MODELS AND COMMANDS *************************

.MODEL SMOD VSWITCH(RON=0.01)

.MODEL DMOD D

.TRAN .1MS 50MS 16.67ms 10US UIC


.FOUR 60 I(RA) ; Fourier analysis of line current

.PROBE

.OPTIONS NOPAGE ITL5=0

.END

_____________________________________________________________________________
________

5-19) The PSpice input file from Example 5-4 is used for this simulation. In Probe, enter the
expression 3*AVG(W(RA)) to get the total three-phase average power in the load, resulting in
368 W. Switch S1 conducts when the current in phase A is positive, and S4 conducts when the
current is negative.

_____________________________________________________________________________
________

5-20) The smallest value of α is 120°. The conduction angel must be less than for equal to 60°.
The extinction angle is 180°, so α is 120° or greater.

_____________________________________________________________________________
________

5-21)

THREE-PHASE VOLTAGE CONTROLLER -- R-L LOAD

*MODIFIED FOR A DELTA-CONNECTED LOAD

*SOURCE IS Y-CONNECTED (UNGROUNDED)

********************** INPUT PARAMETERS ****************************

.PARAM Vs=480 ; rms line-to-line voltage

.PARAM ALPHA=45 ; delay angle in degrees

.PARAM R=25 ; load resistance (y-connected)

.PARAM L = 1p ; load inductance


.PARAM F=60 ; source frequency

********************** COMPUTED PARAMETERS **************************

.PARAM Vm={Vs*SQRT(2)/SQRT(3)} ; convert to peak line-neutral volts

.PARAM DLAY={1/(6*F)} ; switching interval is 1/6 period

.PARAM PW={.5/F} TALPHA={ALPHA/(F*360)}

.PARAM TRF=10US ; rise and fall time for pulse switch control

*********************** THREE-PHASE SOURCE **************************

VAN 1 0 SIN(0 {VM} 60)

VBN 2 0 SIN(0 {VM} 60 0 0 -120)

VCN 3 0 SIN(0 {VM} 60 0 0 -240)

***************************** SWITCHES ********************************

S1 1 8 18 0 SMOD ; A-phase

D1 8 4 DMOD

S4 4 9 19 0 SMOD

D4 9 1 DMOD

S3 2 10 20 0 SMOD ; B-phase

D3 10 5 DMOD

S6 5 11 21 0 SMOD

D6 11 2 DMOD

S5 3 12 22 0 SMOD ; C-phase

D5 12 6 DMOD

S2 6 13 23 0 SMOD

D2 13 3 DMOD

***************************** LOAD **********************************

RA 4 4A {R} ;

LA 4A 2 {L}

RB 5 5A {R} ;

LB 5A 3 {L}

RC 6 6A {R} ;

LC 6A 1 {L}

************************* SWITCH CONTROL *****************************


V1 18 0 PULSE(-10 10 {TALPHA} {TRF} {TRF} {PW} {1/F})

V4 19 0 PULSE(-10 10 {TALPHA+3*DLAY} {TRF} {TRF} {PW} {1/F})

V3 20 0 PULSE(-10 10 {TALPHA+2*DLAY} {TRF} {TRF} {PW} {1/F})

V6 21 0 PULSE(-10 10 {TALPHA+5*DLAY} {TRF} {TRF} {PW} {1/F})

V5 22 0 PULSE(-10 10 {TALPHA+4*DLAY} {TRF} {TRF} {PW} {1/F})

V2 23 0 PULSE(-10 10 {TALPHA+DLAY} {TRF} {TRF} {PW} {1/F})

************************ MODELS AND COMMANDS *************************

.MODEL SMOD VSWITCH(RON=0.01)

.MODEL DMOD D

.TRAN .1MS 50MS 16.67ms 10US UIC

.FOUR 60 I(RA) ; Fourier analysis of line current

.PROBE

.OPTIONS NOPAGE ITL5=0

.END

40A

Ia

0A

SEL>>
-40A
I(RA)
50A

Source A current

0A

-50A
15ms 20ms 25ms 30ms 35ms 40ms 45ms 50ms
- I(VAN)
Time

_____________________________________________________________________________
________

5-22) The PSpice circuit file modification must include a very large resistor (e.g., one
megaohm) connected between the neutral of the load to ground to prevent a “floating node”
error because of the series capacitor. The steady-state phase A current has two pulses for each
of the switches, assuming that the gate signal to the SCRs is continuously applied during the
conduction interval. The rms current is approximately 5.52 A. The total average power for all
three phases is approximately 1.28 kW. The THD for the load current is computed as 140% for
harmonics through n = 9 in the .FOUR command. However, the current waveform is rich in
higher-order harmonics and the THD is approximately 300% for n = 100. It should be noted
that this load is not conducive for use with the voltage controller because the load voltage will
get extremely large (over 5 kV) because of stored charge on the capacitor.

40A
S1 S1
(1.0000,5.5229)

0A

Phase A current
SEL>> S4 S4
-40A
I(RA) RMS(I(RA))
2.0KW

(1.0000,1.2811K)

1.0KW

Total average power

0W
0.980s 0.984s 0.988s 0.992s 0.996s 1.000s
AVG(W(RA))*3
Time

_____________________________________________________________________________
________

5-23) With the S1-S4 switch path open, the equivalent circuit is as shown. The current in phase
A is zero, so the voltage across the phase-A resistor is zero. The voltage at the negative of V14 is
then Vn, and the voltage at the positive of V14 is Va. The voltage across the phase B resistor is
half of the voltage from phase B to phase C, resulting in

Vb − Vc Vb + Vc
Vn = Vb − =
2 2
Therefore,
Vb + Vc
V14 = Va − Vn = Va −
2
CHAPTER 6 SOLUTIONS
5/17/10

6-1)

Po = Vo I o = Vo I s ; Ps = Vs I s

Po Vo I o Vo
= = =
Ps Vs I s Vs

6-2)

P 100
Io = = = 3.33 A.
Vo 30
Po 100
a) Ps = Vs I o = (100)(3.33) = 333 W .;  = = = 30%
Ps 333
b) PQ = VCE I o = (70)(3.33) = 233 W .
1 yr.=8760 Hr.; W = (233)(8760)=2044 kW-Hr,
c) e.g., @10 cents/kW-Hr, cost = $204.40/yr.

6-3)

a) Vo = Vs D = (100)(0.6) = 60 V .
b) Vo ,rms = Vm D = (100) 0.6 = 77.5 V (see Example 2-4)
Vo2,rms77.52
c) P = = = 600 W .
R 10
d ) Results are not dependent on frequency.

6-4)
a) Vo = Vs D = (24)(0.65) = 15.6 V .
Vo 15.6
b) I L = I R = = = 1.56 A.
R 10
V 15.6 1
iL = o (1 − D)T = −6
(1 − 0.65) = 2.18 A.
L 25(10) 100, 000
i 2.18
I L ,max = I L + L = 1.56 + = 2.65 A.
2 2
i 2.18
I L ,min = I L − L = 1.56 − = 0.47 A.
2 2

Vo (1 − D) 15.6(1 − 0.65)
c) Vo = = = 0.182
8LCf 2
8(25)(10) −6 (15)(10) −6 (100, 000) 2
Vo
or = 1.17%
Vo

6-5)

a ) Vo = Vs D = 9 V .
b) I L = 1.8 A.; iL = 2.4 A.
iL
I L ,max = I L + = 3.0 A.
2
i
I L ,min = I L − L = 0.6 A.
2
Vo
c) = 0.44%
Vo

6-6)
Vo
a) D = = 0.5
Vs
Po 125
b) I L = I R = = = 5 A.
Vo 25
iL V
I L ,max = 6.25 A.  = 1.25; iL = 2.5 A. = o (1 − D)T
2 L
V 25 1
L = o (1 − D)T = (1 − .5) = 50  H .
iL 2.5 100, 000
Vo 1− D
c)  = 5% = .005 =
Vo 8LCf 2
1− D 1 − .5
C= = = 25  F .
 Vo  2 8(.005)(50)(10)−6 (100, 000)2
8  Lf
 Vo 

6-7)
Vo 1.5
a) D = = = 0.25
Vs 6
2
Vo 1.5  0.5625 / 2 
b) average : I L = I R = = = 0.5 A. rms : I L ,rms = 0.52 +   = 0.526 A.
R 3  3 
iL = 0.5625
 1 1− D  1 1 − .25 
peak : I L,max = Vo  +  = 1.5  + −6  = 0.781 A.
 R 2 Lf   3 2(5)(10) (400,000) 
 1 1− D 
I L,min = Vo  −  = 0.219 A.
 R 2 Lf 
VI 1.5(0.5)
c) Ps = Po  Vs I s = Vo I R  I s = o R = = 0.125 A.
Vs 6
d ) I D ,max = I L,max = 0.781 A.
I D = I o − I s = 0.5 − 0.125 = 0.375 A.

6-8)

Po 25
Io = I L = = = 1.25 A.
Vo 30
Vo 20
D= = = 0.667
Vs 30
iL
I L ,min = (0.25)(1.25) = 0.31 A. = I L −
2
iL = ( I L − I L ,min )2 = (1.25 − 0.31)2 = 1.88 A.
Vo
iL = (1 − D)T
L
V 1 20 1
L = o (1 − D) = (1 − .667 ) = 89  H
iL f 1.88 40000
6-9)

(1 − D) R
Lmin =
2f
Vo 20 20
D= ; Dmax = = 0.4; Dmin = = 0.33
Vs 50 60
Po 75 125
IL = IR = ; I R ,min = = 3.75 A.; I R ,max = 6.25 A.
Vo 20 20
Vo 202 202
R= ; Rmax = = 5.33 ; Rmin = = 3.20 
P 75 125
(1 − Dmin ) Rmax (1 − .33)(5.33)
Lmin = = = 17.76  H
2f 2(100, 000)

6-10)

(1 − D)( R)
Lmin = f = 200 kHz
2f

Vo=5 V

Vs, V D I, A. R, Ω Lmin, µH

10 0.5 0.5 10 12.5


10 0.5 1.0 5 6.25

15 1/3 0.5 10 16.7 (worst case, D = 1/3, R


= 10)
15 1/3 1.0 5 8.33

(1 − Dmin ) Rmax
L=
2f

 1
 1 −  (10)
=
3
Lmin = 16.67  H
2(200 k )
6-11) Example design:
Vo 15
D= = = 0.3125
Vs 48

Let f = 100 kHz ( for example)

V   15 
Let iL = 40% of I L = 0.40  o  = 0.40  8  = 0.75 A
R   

L=
(Vs − Vo ) D = ( 48 − 15) 0.3125 = 137.5  H
( iL ) f ( 0.75)100,000
1− D 1 − 0.3125
C= = = 12.5  F
 Vo  8 (150 )10−6 (0.005)100,000
8L  f
 Vo 

Other values of L and C are valid if the inductor current is continuous with margin.

6-12) (Based on the example design in 6-11)

Vmax, switch = Vs = 48 V
Vmax, diode = Vs = 48 V
Imax, switch = ILmax = 1.5 + 0.75/2 = 1.875 A

Vo Io 15 (1.875 )
Iavg, switch = = = 0.586 A
Vs 48

DT
1
Irms, switch = i = 1.06A (numerically)
2
L (t)d t
T 0

Imax,diode = ILmax = 1.875 A


Iavg,diode =IL- Iavg,switch = 1.875 – 0.586 = 1.289 A

T
1
Irms,diode =
T DT 
i 2L (t)d t = 1.56A (numerically)
6-13) Example design:

Vo 15
D= = = 0.625
Vs 24

Let f = 400 kHz ( for example)

Let iL = 40% of I L = 0.40 ( 2 ) = 0.8 A

L=
(Vs − Vo ) D = ( 24 − 15) 0.625 = 17.6  H
( iL ) f ( 0.8) 400,000
1− D 1 − 0.625
C= = = 1.67  F
 Vo  8 (17.6 )10−6 (0.01)400,000
8L  f
 Vo 

6-14) Example design:


Vo 12
D= = = 0.667
Vs 18

Let f = 200 kHz ( for example)


Po 10W
I L = Io = = = 0.833 A
Vo 12V

Let iL = 40% of I L = 0.40 ( 0.833) = 0.333 A

L=
(Vs − Vo ) D = (18 − 12 ) 0.667 = 60  H
( iL ) f ( 0.333) 200,000
1− D 1 − 0.667
C= = = 3.5  F
 Vo  −6  0.1 
8L  f 8 ( 60 )10  12  200,000
 Vo   

Other values of L and C are valid if the inductor current is continuous with margin.

6-15)

n = 1  V1 = 30.27
Using ac circuit analysis, Vo1 = 0.048 V = 2(0.048) = 0.096 V p − p
0.096 0.096
= = 0.48%
Vo 20
Vo
Using Eq. 6 -16, = 0.469%
Vo

The output voltage is mainly the dc term and the first ac term.
6-16)

a ) rC = 0.5 , iL = 2.88 A = iC


Vo ,ESR = iC rC = 2.88(0.5) = 1.44 V .
Vo 1.44
= = 8%
Vo 18
Vo
b)  0.5%
Vo
Vo 0.005(18)
Vo  Vo ,ESR = iC rC  rC = = = 0.031 
iC 2.88
50(10) −6 50(10) −6 50(10)−6
rC =  C= = = 1600  F .
C rC 0.031

6-17)
Vs 20
a ) Vo = = = 50 V .
1 − D 1 − .6
Vs 20
b) I L = = = 10 A.
(1 − D ) R (1 − .6) 2 (12.5)
2

Vs V DT 20 20(.6) / (200,000)
I max = + s = + = 13 A.
(1 − D ) R
2
2L (1 − .6) (12.5)
2
2(10)(10) −6
Vs V DT
I min = − s = 7.0 A.
(1 − D ) R
2
2L
Vo D 0.6
c) = = = 0.6%
Vo RCf 12.5(40)(10) −6 (200,000)
Vo 50
d ) I D = Io = = = 4.0 A.
R 12.5

6-18)

Inductor current: (see Example 2-8)


2 2
 I / 2   4.61/ 2 
I L ,rms = I +  L  = 102 + 
2
L  = 10.09 A.
 3   3 

Capacitor current: (define t=0 at peak current)


1/2
 1  10  s 25  s

=   − + +  −  = 4.97 A.
5 2 2
I C ,rms ( 4.61(10) t 8.3) dt ( 4) dt
 25(10)  0 
−6
10  s 

6-19)

Vs V 5
Vo =  D = 1 − s = 1 − = 0.667
1− D Vo 15
Vo2 152
R= = =9 
25 25
Vs 5
IL = = = 5 A.
(1 − D) R (1 − .667) 2 (9)
2

I L ,min = 0.5(5) = 2.5 A.  I L = 5 A.


Vs DT 5(.667) / 300
I L = = = 2.22  H
I L 5
D 0.667
From Eq. 6 − 27, C = = = 24.7  F .
 Vo  9(.01)(300, 000)
R  f
 Vo 
6-20) Example design:

Vs 12
D =1− = 1 − = 0.333
Vo 18

Vo2 182
R= = = 16.2 
P 20
Vs 12
IL = = = 1.67 A
(1 − D ) 2
R (1 − .333)2 16.2
Let f = 200 kHz

Let iL = 40% of I L = 0.4 (1.67 ) = 0.667 A

Vs D 12 ( 0.333)
L= = = 30  H
( iL ) f (0.667)200,000
( Lmin for continuous current = 6  H )

D 0.333
C= = = 20.6  F
 V  16.2 ( 0.005 ) 200,000
R o  f
 Vo 

6-21)

0.6
Using C = 48  F , R = 50 , ton = 0.6T = = 24  s
25000
vo (t ) = Vo ,max e − t / RC
vo (24  s ) = Vo ,max e −24/[(50)(48)] = Vo ,max ( 0.99005)
Vo ,max − vo (24  s ) = Vo = Vo,max − 0.99005Vo,max  0.01Vo,max
Vo
= 0.01 = 1%
Vo
6-22)

6-23)

 D   0.6 
a ) Vo = −Vs  = − 12 1 − 0.6  = −18 V .
1 − D 
Vs D 12(.6)
b) Eq. 6 − 31: I L = = = 4.5 A.
R(1 − D) 2
(10)(1 − .6) 2
Vs D V DT 12(.6) / 200, 000
I L ,max = = s = 4.5 + = 6.3 A.
R(1 − D) 2
2L 2(10)(10) −6
Vs D V DT
I L ,min = − s = 2.7 A.
R(1 − D) 2
2L
Vo D 0.6
c) = = = 0.015 = 1.5%
Vo RCf 10(20)(10) −6 (200, 000)
6-24)

Inductor current: (see Example 2-8)

2 2
 I / 2   3.6 / 2 
I L,rms = I +  L  = 4.52 + 
2
L  = 4.62 A.
 3   3 

Capacitor current: For convenience, redefine t = 0 at the peak current. The current is then
expressed as

( )
iC ( t ) = 4.5 − 1.8 106 t A for 0  t  2 s

= −1.8 A for 2 s  t  5 s
T
= 5 (10−6 ) = 5 s
1 2 1 1
T 0
I rms = i (t )dt T= =
f 200, 000

1/2
 1  2s 5s

( )
2


  
−6
I C ,rms =  4.5 − 1.8 10  dt + ( −1.8) 2
dt  = 2.30 A.
 5(10)  0 
−6
2s 

6-25)

Vo 36
a ) From Eq. 6-48, D = = = 0.6
Vs + Vo 24 + 36
Vs D 24(.6)
IL = = = 9 A.  I L ,min = 0.4(9) = 3.6 A.
R(1 − D) 10(1 − .6) 2
2

I L = 2(9 − 3.6) = 10.8 A.


Vs DT 24(.6)
From Eq. 6-28, L = = = 13.3  H
I L 10.8(100, 000)

D 0.6
b) From Eq. 6-36, C = = = 120 F
 Vo  10(0.005)(100, 000)
R f
 Vo 

6-26) Example design:


Vo 50
Using Eq. (6-48), D = = = 0.556
Vs + Vo 40 + 50

P 75
Using Eq. (6-49), IL = = = 3.375 A.
Vs D 40 ( 0.556 )

Vo2 502
R= = = 33.3 
P 75
Letting f = 100 kHz (designer's choice),

(1 − D )2 R (1 − 0.556 )2 33.3
Lmin = = = 32.9  H
2f 2 (100, 000 )

Choose L at least 25% larger than Lmin (41  H). A common practice is to select L such that
i L = 40% of I L = 0.40 ( 3.375) = 1.35 A. Using Eq. (6-45),

Vs D 40 ( 0.556 )
L= = = 165  H
iL f 1.35 (100, 000 )

D 0.556
Using Eq. (6-54), C = = = 16.7  F
 V  33.3 ( 0.01)(100, 000 )
R o  f
 Vo 
6-27) Example design:

Vo 15
Using Eq. (6-48), D = , D= = 0.556 for the 12-V source,
Vs + Vo 12 + 15

15
and D = = 0.455 for the 18-V source.
18 + 15

(1 − D )2 R
Using Lmin = , the worst case is for D = 0.455 for the 18-V source.
2f
(1 − 0.455)2 15
Letting f = 100 kHz (designer's choice), Lmin = = 22.3  H
2 (100, 000 )

Choose L at least 25% larger than Lmin (28  H).

Alternatively, a common practice is to select L such that i L = 40% of I L .

Vo2 152
Il = = = 1.83 A
Vs RD 18 (15 )( 0.455 )

iL = 0.40 (1.83) = 0.73 A. Using Eq. (6-45),

Vs D 18 ( 0.455 )
L= = = 112  H (100  H will be fine)
iL f 0.73 (100, 000 )

D
Using Eq. (6-54), C = , so base C on D = 0.556, (12-V source):
 V 
R o  f
 Vo 
0.556
C= = 37  F
15 ( 0.01)(100, 000 )
6-28) Using the equations

Vo
D=
Vs + Vo

Vo2
R=
P

Lmin =
(1− D) R
2

2f
P
IL =
Vs D

D
C=
 V 
R o  f
 Vo 

and using f = 100 kHz (designer’s choice), results are shown in the table.

Vs, (V) P (W) D R (Ω) Lmin (µH) IL (A) C (µF)

10 10 0.545 14.4 14.9 1.83 37.9

10 15 0.545 9.6 9.9 2.75 56.8

14 10 0.462 14.4 20.9 1.55 32.1


14 15 0.462 9.6 13.9 2.32 48.1

The value of L should be based on Vs = 14 V and P = 10 W, where Lmin = 20.9 µH. Select the
value of L at least 25% larger than Lmin (26.1 µF). Using another common criterion of ΔiL =
40% of IL, again for 14 V and 10 W, L = 104 µH.

The value of C is 56.8 µF for the worst case of Vs = 10 V and P = 10 W.


6-29)

 D   0.6 
Vo = −Vs   = −12  = −18 V .
1 − D  1 − 0.6 
Po Vo2 R 27
I L2 = = = = 1.5 A.
−Vo −Vo 18
Po 27
I L1 = = = 2.25 A.
Vs 12
Vs D 12(.6)
iL1 = = = 0.14 A.
200 (10 ) (250,000)
−6
Lf
Vs D
iL 2 = = 0.29 A.
L2 f

6-30)

1 1
D= = = 0.333
Vs 20
1− 1−
Vo −10
I L 2 = I o = 1 A.
Vo  10 
I L1 = I L 2 =   (1) = 0.5 A.
Vs  20 
VD VD 20(0.333)
iL1 = s  L1 = s = = 1.33 mH
L1 f iL1 f 0.10(.5)(100,000)
Vs D VD 20(0.333)
iL 2 =  L2 = s = = 0.667 mH
L2 f iL 2 f 0.10(1)(100,000)
6-31) Example design:

Vo D −30
=− = = −1.2  D = 0.5455
Vs 1 − D 25

Po 60 VD 25 ( 0.5445 )
I L2 = = = 2.0 A;  iL 2 = 0.4(2.0) = 0.4 A  L2  s = = 341  F
−Vo 30 iL 2 f 0.4 (100,000 )

Ps 60 Vs D 25 ( 0.5445 )
I L1 = = = 2.4 A;  iL1 = 0.4(2.4) = 0.48 A  L1  = = 284  F
Vs 25 iL1 f 0.48 (100,000 )

Let f = 100 kHz (designer's choice).


1− D 1 − 0.5455
C2  = = 1.67  F
 Vo 
  8 L2 f
2 ( 0.01)( ( ))
8 341 10 −6
( 100,000 ) 2

 Vo 
VC1 = Vs − Vo = 25 − ( −30 ) = 55 V  vC1 = 0.05 ( 55 ) = 2.75 V

Using R = Vo2 / P = ( −30 ) / 60 = 15 ,


2

Vo D 30 ( 0.5455 )
C1  = = 3.97  F
Rf vC1 15 (100,000 ) 2.75
6-32)

Vo 12
D= = = 0.706
Vo + Vs 12 + 5

Vo 2 122
I L1 = = = 7.2A
Vs R 5(4)

Vs D (5)(0.706)
i L1 = = = 3.53A
L1f 10(10)-6 (100,000)

3.53
I L1,max = 7.2 + = 8.96A
2
3.53
I L1,min = 7.2 − = 5.44A
2
Vo 12
I L2 = = = 3A
R 4
Vs D (5)(0.706)
i L1 = = = 1.765A
L2f 20(10)-6 (100,000)

1.765
I L2,max = 3 + = 3.88A
2
1.765
I L2,min = 3 − = 2.12A
2
6-33)

Vs D 3.3(.7)
Vo = = = 7.7V
1 − D 1 − .7
Vo 2 7.7 2
I L1 = = = 3.6A
Vs R 3.3(5)

Vs D (3.3)(0.7)
i L1 = = = 1.925A
L1f 4(10)-6 (300,000)

1.925
I L1,max = 3.6 + = 4.56A
2
1.925
I L1,min = 3.6 − = 2.64A
2
Vo 7.7
I L2 = = = 1.54A
R 5
Vs D (3.3)(0.7)
i L1 = = = 0.77A
L 2 f 10(10)-6 (300,000)

0.77
I L2,max = 1.54 + = 1.925A
2
0.77
I L2,min = 1.54 − = 1.155A
2
Vo D (7.7)(.7)
VC1 = VC2 = = = 0.0719V
RCf 5(50)(10)−6 (300,000)

6-34) Equation (6-69) for the average voltage across the capacitor C1 applies:

VC1 = Vs .

When the switch is closed, the voltage across L2 for the interval DT is

v L2 = v C1

Assuming that the voltage across C1 remains constant at its average value of Vs

vL2 = VC1 = Vs (switch closed)


When the switch is open in the interval (1 - D)T,

vL2 = −Vo (switch open)

Since the average voltage across an inductor is zero for periodic operation,

(v L2, sw closed ) ( DT ) + ( v L2, sw open ) (1 − D ) T = 0


Vs ( DT ) − Vo (1 − D ) T = 0
resulting in

 D 
Vo = Vs  
1− D 

6-35)

Vo 2 62
I L1 = Is = = = 1.2A
Vs R (15)(2)

Vo 6
D= = = 0.286
Vo + Vs 6 + 15

Vs D (15)(0.286)
L1 = = = 35.7H
( i L1 )f 0.4(1.2)250,000

Vo 6
I L2 = I o = = = 3A
R 2
Vs D (15)(0.286)
L2 = = = 14.3H
( i L2 )f 0.4(3)250,000

VC2 = Vo = 6

Vo D D 0.286
VC2 = Vo = or C2 = = = 28.6F
RC2f  Vo  2(.02)250,000
R f
 Vo 
C1 = C2 = 28.6F
6-36)

Vo 2.7
D= = = 0.231
Vo + Vs 2.7 + 9

2.7
R= = 2.7
1
Vo 2 2.72
I L1 = = = 0.30A
Vs R 9(2.7)

Vs D 9(0.231)
L1 = = = 57.7H
( i L1 ) f 0.4(0.30)300,000
I L2 = I o = 1A

Vs D 9(0.231)
L2 = = = 14.2H
( i L2 ) f 0.4(1)300,000

6-37)

iC = I L ,max = 2.7 A.


VO , ESR = iC rC = (2.7)(0.6) = 1.62 V .
VO , ESR 1.62
= = 0.054 = 5.4%
VO 30
Worst case : VO = VO ,C + VO , ESR = 0.3 + 1.63 = 1.92 V . = 6.4%

6-38)

Switch closed : vL = Vs − VQ
Switch open : vL = VO − VD
avg (vL ) = 0 : (Vs − VQ ) DT + (VO − VD )(1 − D)T = 0
 D 
 VO = VD − (Vs − VQ )  
 1− D 
6-39)

(1 − D) R L 2 f 97.5(10) −6 (2)(40000)
a ) Lmin −  Rmax = = = 12.5 
2f 1− D 1 − .375
b) For R=20 Ω, current is discontinuous:
   
   
2D 2(0.375)
Vo = Vs   = 48   = 21.4 V .
 8L   8(97.5)(10) −6 
 D+ D +  0.375 + (0.375) +
2
 
2

 RT   20 / 40000 
 18  Vo  21.4 V .
(1 − D) R (1 − 0.375)(20)
c) Lf min = = = 6.25
2 2
6.25 6.25
Increase Lf min : e.g., Lmin = = 157  H or f min = = 64.1 kHz
40000 97.5 mH

6-40)

2 Lf 2(120)(10) −6 (25000)
a ) Rmax = = = 62.5  for continuous current
D(1 − D) 2 0.6(1−).6) 2
b) For R=100 Ω, the current is discontinuous:
 2 D 2 RT 
 1+ 1+ 
Vo = Vs  L 
 2 
 
 
 2(.6) 2 (100) / 25000 
 1+ 1+ 
120(10) −6
Vo = 12   = 36 V .
 2 
 
 
 
 30  Vo  36 for 25  R  100
4.8 4.8
c) Lf min = 4.8  L  = 192  H or f  = 40 kHz.
25000 120(10) −6
6-41) Discontinuous current for the buck-boost converter: Let DT be the time that the switch
is closed and D1T be the time that the switch is open and the current in the inductor is positive.
For a lossless converter, the output power is the same as the input power.

I D
Ps = Vs I s = Vs  max 
 2 
V DT
I max = s
L
2 2
V DT
Ps = s
2L
2
V
Po = o
R
2 2
Vs D T Vo2
=
2L R
Vo RT
=D
Vs 2L

6-42) When switches “1” are closed, C1 and C2 are connected in series, each having Vs/2 volts.
When the “1” switches are opened and the “2” switches are closed, Vo = Vs of the source plus
Vs/2 of C1, making Vo = 1.5Vs.

6-43)
20W

p(t) for the MOSFET

10W

0W

(1.000m,405.19m)
Average P = 405 mW

-10W
0.990ms 0.992ms 0.994ms 0.996ms 0.998ms 1.000ms
W(M1) AVG(W(M1))
Time
6-44) Simulate the buck converter of Example 6-1 using PSpice. (a) Use an ideal switch and ideal
diode. Determine the output ripple voltage. Compare your PSpice results with the analytic results
in Example 6-1. (b) Determine the steady-state output voltage and voltage ripple using a switch
with an on resistance of 2 Ω and the default diode model

Using Ron =0.01 for the switch and n=0.01 for the diode, the p-p ripple voltage is 93.83 mV.
93.83/20 = 0.469%, agreeing precisely with the analytical results.

With Ron = 2 ohms, the p-p ripple is 90 mV, with a reduced average value.
6-45)
Note that for each converter topology, the average voltage across each inductor is zero,
and the average current in each capacitor is zero.

Buck Converter:
Show from Eqs. (6-9) and (6-17)

Is
Vo = Vs D and Io =
D

From the averaged circuit of Fig. 6.33b,

Is
I L = Io = i c and Is = Dic  Io =
D
Vap = Vs and Vo = DVap  Vo = DVs

Boost Converter:
Show from Eqs. (6-27) and (6-28) that
Vs
Vo = and Io = Is (1 − D )
1− D

From the averaged circuit of Fig. 6.33c,


Vs
DVap = Vs − Vo and Vap = −Vo  Vo =
1− D
Io = −ic + Dic = ic ( D − 1) and Is = −i c  Io = Is (1 − D )

Buck-Boost Converter:
Show from Eqs. (6-47) and (6-49) and preceding equations that

 D 
Vo = −Vs   and Is = I L D
1− D 

From the averaged circuit of Fig. 6.33d,

IL = ic and Is = Dic  Is = I L D

 D 
Vap = Vs − Vo and Vo = −DVap  Vo = −Vs  
1− D 

Ćuk Converter:
Show from Eqs. (6-59) and (6-61) that

IL1 D  D 
= and Vo = −Vs  
IL2 1 − D 1− D 
From the averaged circuit,
 D 
Vap = Vs − Vo and DVap = −Vo  Vo = −Vs  
1− D 
I L1 D
i c = Di c + I L2 and I L1 = Di c =
I L2 1 − D
CHAPTER 7 SOLUTIONS

4/03/10
7-1)
 D  N2  0.4  1 
a ) Vo = Vs   = 36    = 12 V .
 1 − D  N1  0.6  2 
Vo2 122
b) I Lm = = = 1.67 A.
Vs DR 36(0.4)(6)
Vs D 36(0.4)
iLm = = = 1.44 A.
Lm f 100(10) −6 (100, 000)
iLm
I Lm,max = I Lm + = 2.39 A.
2
i
I Lm,min = I Lm − Lm = 0.947 A.
2
Vo D 12(0.4)
c) Vo = = = 0.16 V .
RCf 6(50)(10) −6 (100, 000)
Vo 0.16
= = 1.33%
Vo 12

7-2)
 D  N2  0.6  1 
a ) Vo = Vs   = 4.5    = 16.9 V .
 1 − D  N1  0.4  0.4 
Vo2 (16.9)2
b) I Lm = = = 7.03 A.
Vs DR 4.5(0.6)(6)
Vs D 4.5(0.6)
iLm = = = 1.08 A.
Lm f 10(10) −6 (250, 000)
iLm
I Lm,max = I Lm + = 7.57 A.
2
i
I Lm,min = I Lm − Lm = 6.49 A.
2
D (0.6)
c) Vo = = = 1.6%
RCf 15(10)(10) −6 (250, 000)

7-3)
 D  N2 N 2 Vo  1 − D  3  1 − .32 
a ) Vo = Vs    =  =   = 0.145
 1 − D  N1 N1 Vs  D  44  .32 
N
or 1 = 6.90
N2
Vo2 32
b) I Lm = = = 0.640 A.
Vs DR 44(0.32)(1)
iLm = 0.4 I Lm = 0.4(0.640) = 0.256 A.
Vs D 44(0.32)
Lm = = = 184  H .
iLm f (0.256)(300, 000)

7-4) Example design


Vs = 24 V . Po = 40 W . Vo = 40 V .
N 2 Vo  1 − D 
=  
N1 Vs  D 
Let D = 0.4 (arbitrary )
then
N 2 40  1 − 0.4  N1
=   = 2.5, or = 0.4
N1 24  0.4  N2
Vo  N2  40
I Lm =  = (2.5) = 4.17 A
(1 − D) R  N1  (1 − 0.4)40
Vo2 402
where R = = = 40 
P 40
Let iLm = 40% of I Lm = 0.4(4.16) = 1.67 A
Let f = 100 kHz
VD 24(0.4)
Lm = s = = 57.6  H
iLm f 1.67(100, 000)
D 0.4
C= = = 20  F
 Vo  40(.005)(100, 000)
R f
 o 
V

7-5) For continuous current


iLm
I Lm ,min = I Lm −
2
2
V VD
I Lm ,min =0= o − s
Vs DR 2 Lm f
Vo2 2 Lm f 52 (2)(500)(10−6 )40, 000
R= = = 11.7 
(Vs D) 2 [24(0.385)]2
R  11.7   continuous current
R  11.7   discontinuous current
7-6) Switch is closed for DT, current returns to zero at t = tx:
Vs DT
I Lm ,max =
Lm
diLm −Vo  N1 
Switch open : =  
dt Lm  N 2 
−Vo  N1  −Vo  N1 
t
Vs DT
iLm (t ) =    d  + iLm ( DT ) =   (t − DT ) +
Lm  N 2  DT Lm  N 2  Lm
−Vo  N1  Vs DT
I LM (t = t x ) = 0 =   (t x − DT ) +
Lm  N 2  Lm
Vs DT  N 2 
tx =   + DT
Vo  N1 

7-7)
N 
a) Vo = Vs D  2  = 100(.35)(1) = 35 V .
 N1 
Vo 1− D 1 − .35
= = = 0.16%
Vo 8LxCf 2 −6
8(70)(10) 33(10) −6 (150, 000) 2
Vo 35
b) I Lx = = = 1.75 A.
R 20
 N   D .35
iLx = Vs  2  − Vo  = [100(1) − 35] −6
= 2.17 A.
  N1   Lx f (70)(10) (150, 000)
2.17
I Lx ,max = 1.75 + = 2.83 A.
2
2.17
I Lx ,min = 1.75 − = 0.67 A.
2
V DT 100(.35)
c) iLm = s = 0.233 A.
Lm 1(10) −3150, 000
d ) isw = I1 + iLm = i pri
I sw,max = I Lx ,max (1/1) + I Lm,max = 2.83 + 0.233 = 3.06 A.

7-8)
N  1
a) Vo = Vs D  2  = 170(0.3)   = 5.1 V .
 N1   10 
Vo 1− D 1 − 0.3
= = = 0.175%
Vo 8LxCf 2 −6
8(20)(10) 10(10) −6 (500, 000) 2
b)
The currents in the converter are shown below. The winding currents are for the
windings in the ideal transformer model, not the physical windings. The
physical primary winding current is the sum of winding #1 and Lm currents.
Vs DT 170(0.3)
c) iLm = = = 0.3 A.
Lm 340(10) −6 500, 000
1 1
Peak energy in Lm : Wmax = Lm ( iLm ) = (340)(10) −6 (0.3) 2 = 15.3  J
2

2 2
W
P= = Wf = 15.3(10) −6 (500, 000) = 7.65 W .
T

7-9)
N   N  V 50
a) Vo = Vs D  2   D  2  = o = = 0.625
 N1   N1  Vs 80
N
If 3 = 1, then D  0.5
N1
N 2 0.625 N
Let D = 0.3, then = = 2.08 or 1 = 0.48 (not unique)
N1 0.3 N2
Vo2 502 V 50
R= = = 10 ; I Lx = o = = 5 A.
P 250 R 10
V (1 − D) 50(1 − .3)
iLx = o = = 3.5 A.
Lf 100(10) −6100, 000
3.5
I Lx ,min = 5 − = 3.25 A.  0  continuous current
2
Vo 1− D 1 − 0.3
b) = = = 0.058%
Vo 8LxCf 2 −6
8(100)(10) 150(10) −6 (100, 000) 2

7-10)
N  1
a) Vo = Vs D  2  = 100(0.25)   = 5 V .
 N1  5
using Lx = 20  H ,
Vo 1− D 1 − 0.25
= = = 0.33%
Vo 8LxCf 2 −6
8(20)(10) 10(10) −6 (375, 000) 2
b)
The currents in the converter are shown below. The winding currents are for the
windings in the ideal transformer model, not the physical windings. The
physical primary winding current is the sum of winding #1 and Lm currents.
Vs DT 100(0.25)
c) iLm = = 0.20 A.
Lm 333(10) −6 375, 000
1 1
Lm ( iLm ) = (333)(10) −6 (0.2) 2 = 6.66  J
2
Peak energy in Lm : Wmax
2 2
W
P= = Wf = 6.66(10) −6 (375, 000) = 2.5 W .
T

7-11)
N   N  V D 125(0.3)
a) Vo = Vs D  2    1  = s = = 0.75
 1
N  2
N V o 50
V 50
b) I Lx = o = = 2 A.; I Lx ,min = (0.4)(2) = 0.8 A.; iLx = 2(2 − 0.8) = 2.4 A.
R 25
V (1 − D)T V (1 − D)T 50(1 − 0.3)
iLx = o  Lx = o = = 58.3  H .
Lx iLx 2.4(250, 000)
Vo 1− D 1− D 1 − 0.3
c) =  C= = −6
= 4.8  F .
Vo 8LxCf 2  Vo  2 8(58.3)(10) (0.005)(250, 000) 2
8Lx  f
 Vo 

7-12)
N1
Let = 1, then D  0.5
N3
Let D = 0.35, then
N1 Vs D (170)(.35)
= = = 1.2396
N2 Vo 48
N1
Rounding , let = 1.25
N2
Vo  N1   48 
Then D =  =  (1.25) = 0.353
Vs  N 2   170 
Let f = 200 kHz , and design for iLx = 40% of I Lx
Vo (1 − D) 48(1 − 0.353)
Lx = = = 124  H
0.4 I Lx f 0.4(3.125)200, 000
Po 150
where I Lx = I o = = = 3.125 A.
Vo 48
Alternatively, solving for the minimum Lx for continuous current ,
iLx Vo Vo (1 − D)
I Lx ,min = 0 = I Lx − = −
2 R 2 Lx f
(1 − D) R (1 − 0.353)(15.36)
Lx ,min = = = 24.9  H
2f 2(200, 000)
Vo2 482
where R = = = 15.36 
Po 150
Lx must be greater than 24.9  H with margin, (e.g ., 25% greater )
making Lx  31  H

Using Lx = 124  H ,
1− D 1 − 0.353
C = = 1.63  F
 Vo  2 8(124)(10) −6 (0.01)(200, 000) 2
8Lx  f
 Vo 

7-13)
150  Vs  175 V .
Vo = 30 V .
20  Po  50 W  0.667  I o  1.667 A.
Example design :
N
Let 1 = 1, then D  0.5
N3
Let D = 0.3 for Vs = 150 V .
N1 Vs D (150)(0.3)
Then = = = 1.5
N2 Vo 30
Vo  N1   30 
For Vs = 175 V ., D =  =  (1.5) = 0.35
Vs  N 2   175 
 0.3  D  0.35,
which is an acceptable range of D. Other choices are possible.
Using the design criterion of iLx = 40% of I Lx ,
Vo (1 − D)
Lx =
0.4 I Lx f
The worst case is for the smallest D and the smallest I Lx .
Letting f = 250 kHz (arbitrary ),
30(1 − 0.3)
Lx = = 315  H
0.4(0.667)(250, 000)
1− D 1 − 0.3
C = −6
= 2.22  F
 Vo  2 8(315)(10) (0.002)(250, 000)2
8 Lx  f
 Vo 

7-14)
The current in the physical primary winding is the sum of iL1 and iLm in the model. The
physical currents in windings 2 and 3 are the same as in the model.
7-15)
N 
a) Vo = 2Vs  s  D = 2(50)(0.5)(0.35) = 17.5 V .
N 
 p
V 17.5
b) I Lx = o = = 2.19 V .
R 8
V 17.5
iLx = o (0.5 − D)T = (0.5 − 0.35)150, 000 = 0.29 A.
Lx 60(10) −6
iLx 0.29
I Lx ,max = I Lx + = 2.19 + = 2.33 A.
2 2
i 0.29
I Lx ,min = I Lx − Lx = 2.19 − = 2.04 A.
2 2
Vo 1 − 2D 1 − 2(0.35)
c) = = = 0.018%
Vo 32 Lx Cf 2
32(60)(10) −6 39(10) −6 (150, 000) 2

7-16)
7-17)
diLm
Sw1 closed : vP1 = Vs = 50 = vLm = Lm
dt
diLm VLm 50
= = = 25(10)3 A / s
dt Lm 2(10) −3
0.35
For DT = = 2.33  s  iLm = 25(10)3 2.33(10) −6 = 58.3 mA.
150, 000
Sw2 closed : vP 2 = −Vs , VP1 = −Vs
diLm
= −25(10)3 A / s
dt
diLm
Both switches open : VP1 = 0  =0
dt

7-18) The input voltage vx to the filter is Vs(Ns / Np) when either switch is on, and vx is zero
when both switches are off. (See Fig. 7-8.) The voltage across Lx is therefore
N 
vLx = Vs  s  0  t  DT
 Np
 
= −Vo DT  t  T / 2
 N  T   1 
VLx = Vs  s  DT − Vo  − DT    =0
  N p  2    T / 2 
N 
Vo = 2Vs  s  D
 Np
 
7-19)
 Vs   N s   24  1 
Vo =    =    = 17.1 V .
 2(1 − D)   N p   2(1 − 0.65)   2 
Vo2 17.12
I Lx = = = 1.22 A.
Vs R 24(10)
N 
VSw,max = 2Vs  p  = 2(24)(2) = 96 V .
 Ns 
7-20)
 Vs   N s  V N p / N s 50
Vo =     o = =
 2(1 − D)   N p
 Vs 2(1 − D) 30
Let D = 0.7 ( D  0.5)
N s  50 
=   (2)(1 − 0.7) = 1.0
N p  30 
Vo2 502 Vo2 502
R= = 62.5; I Lx = = = 1.33 A.
Po 40 Vs R 30(62.5)
7-21)
Ps = Po
Vs I Lx = Vo I o
 N   2(1 − D)T 
I x = I o = I Lx  p   
 Ns   T 
 N  
Vs I Lx = Vo  I Lx  p  2(1 − D) 
  Nd  
 N  1 
Vo = Vs  S  
 N   2(1 − D) 
 p

7-22)
The simulation is run using a Transient Analysis with a restricted time of 3 to 3.02 ms,
representing two periods of steady-state operation. The steady-state output voltage has
an average value of approximately 30 V and peak-to-peak ripple of approximately 600
mV, ignoring the negative spike. The average transformer primary and secondary
currents are 912 mA and 83.5 mA, respectively. The output voltage is lower than the
predicted value of 36 V because of the nonideal switch and diode, mostly from the
switch. The output voltage ripple is 2%, matching the predicted value. The converter
would operate much better with a switch that has a lower on resistance.
30.5V
Output voltage
(3.0041m,30.295)

(3.0200m,30.057)

30.0V

(3.0141m,29.697)

29.5V
3.000ms 3.004ms 3.008ms 3.012ms 3.016ms 3.020ms
V(Output) AVG(V(Output))
Time

4.0A

Primary current

2.0A (3.0200m,912.072m)

0A
I(TX1:1) AVG(I(TX1:1))
200mA

0A
(3.0200m,83.489m)
Secondary current

SEL>>
-200mA
3.000ms 3.005ms 3.010ms 3.015ms 3.020ms
I(TX1:3) AVG(I(TX1:3))
Time

7-23)
Using a nonideal switch and diode produces lower values for the currents. For iLx, the
maximum, minimum, and average values in PSpice are 1.446 A, 0.900 A, and 1.17 A,
compared to 1.56 A, 1.01 A, and 1.28 A, respectively. However, the peak-to-peak
variation in iLx in PSpice matches that of the ideal circuit (0.55 A).

2.0A
(3.6286m,900.720m)
(3.6114m,1.4463)
SEL>>
0A
I(Lx)
2.0A

0A (3.6114m,1.5068)
(3.6287m,603.330m)
-2.0A
I(L1)
2.0A

0A (3.6114m,1.4463)
(3.6287m,903.647m)

-2.0A
-I(L2)
1.0A

0A (3.6115m,539.288m)

-1.0A
3.60ms 3.61ms 3.62ms 3.63ms 3.64ms 3.65ms 3.66ms
I(L3)
Time
7-24)
Design for θco= -210° and a gain of 20 dB for a cross over frequency of 12 kHz.
From Eq. 7 − 85, K = 3.73 : co = 2 12000 = 75400 radis
co75400
z = = = 20200 rad / s
K 3.73
R
For gain = 20 dB, 2 = 10
R1
Let R1 = 1 k , R2 = 10 k 
1 1
C1 = = 4.95 nF ; C2 = = 355 pF
R2z  p R2
7-25)
 comp = 45 − (−105) = 150
   150 
K = tan  comp  = tan   = 3.73
 2   2 
Gain = 15 dB + 9.5 dB = 24.5 dB
G = 1024.5/20 = 16.8
R2
= 16.8
R1
Let R1 = 1 k  and R2 = 16.8 k 
K 3.73
C1 = = = 7.07 nF
2 f co R2 2 (5000)(16,800)
1 1
C2 = = = 508 pF
K 2 f co R2 3.73(2 5000)(16,800)

7-26)
Using Vs = 6 V as in Example 7-8, the frequency response of the open-loop system
shows that the crossover frequency is approximately 16.8 kHz. The phase angle at the
crossover frequency is 17°, which is much less than the desired value of at least 45°.
Therefore, the system does not have the desired degree of stability.

120

Phase

80

Magnitude (dB)
40

(16.814K,16.866)

(16.814K,48.439m)
0

-40
10Hz 100Hz 1.0KHz 10KHz 100KHz
DB(V(error)) P(V(error))
Frequency
7-27)
a) A frequency response of the circuit yields Vo ≈ -2.5 dB and θ ≈ 103° at 10 kHz.

40
Magnitude

Phase (10.000K,-2.5181)
-0

-40

-80 (10.000K,-102.646)

-120
1.0Hz 10Hz 100Hz 1.0KHz 10KHz 100KHz
DB(V(Output)) P(V(Output))
Frequency
b) With Vp = 3, the gain of the PWM function is 20log10(1/3) = -9.54 dB. The
required gain of the compensated error amplifier is then 2.5 + 9.54 = 12.06 dB,
corresponding to a gain magnitude of 4.0. The phase angle of the compensated
error amplifier at the crossover frequency to give a phase margin of 45° is

comp = 45 − (−103) = 148

From 7-75, 7-85, 7-86, and 7-87,

Let R1 =1kΩ, then R 2 = 4 kΩ.

  comp   148 
K = tan   = tan   = 3.49
 2   2 
K 3.49
C1 = = = 13.8 nF
2 f co R2 2 (10,000)(4000)

1 1
C2 = = = 1.14 nF
K 2 f co R2 (3.49)2 (10,000)(4000)

c) Referring to Example 7-9, the PSpice simulation results are shown indicating
a stable control system. The switching frequency was not specified, and 50 kHz
was used here. Use initial conditions for the capacitor voltage at 8 V and the
inductor current at 2 A.
10

Output voltage

Inductor current

step change

0
0s 0.5ms 1.0ms 1.5ms 2.0ms 2.5ms 3.0ms
V(OUTPUT) I(L1)
Time
7-28)
a) The gain at 8 kHz is approximately -2.44 dB, and the phase angle is -100°.

40
Magnitude

(8.0358K,-2.4358)
Phase
-0

-40

-80 (8.0358K,-100.156)

-120
1.0Hz 10Hz 100Hz 1.0KHz 10KHz 100KHz
DB(V(Output)) P(V(Output))
Frequency

b) This design is for fco = 8 kHz. With Vp = 3, the gain of the PWM function is
20log10(1/3) = -9.54 dB. The required gain of the compensated error amplifier is then
2.44 + 9.54 = 11.98 dB, corresponding to a gain magnitude of 3.97. The phase angle of
the compensated error amplifier at the crossover frequency to give a phase margin of
45° is

comp = 45 − (−100) = 145

From 7-75, 7-85, 7-86, and 7-87,


Let R1 =1kΩ, then R 2 = 3.97 kΩ.

  comp   145 
K = tan   = tan   = 3.17
 2   2 
K 3.17
C1 = = = 15.9 nF
2 f co R2 2 (8000)(3970)

1 1
C2 = = = 1.58 nF
K 2 f co R2 (3.17)2 (8000)(3970)

c) Referring to Example 7-9, the PSpice simulation results are shown indicating
a stable control system. The switching frequency was not specified, and 50 kHz
was used here. Use initial conditions for the capacitor voltage at 8 V and the
inductor current at 1.6 A.

10

Output voltage

Inductor current

step change

0
0s 0.5ms 1.0ms 1.5ms 2.0ms 2.5ms 3.0ms
I(L1) V(Output)
Time

If designing for fco = 10 kHz, the gain of the converter is -4.38 dB, and θco = -
98°. R1 = 1k, R2 = 4.97k, C1 = 9.58 nF, and C2 = 1.07 nF.
7-29)
2
  + 90     195 + 90  
2

K =  tan  comp   =  tan    = 8.68


  4    4 
f co = 15 kHz  co = 2 f co = 94, 248 rad / s.
20 log10 (G ) = 15 dB
G = 1015/20 = 5.62
Using Eq. (7 - 112) and letting R1 = 1 k 
GR1
R2 = = 1.91 k 
K
K
C1 = = 16.4 nF
co R2
1
C2 = = 1.89 nF
co R2 K
K
C3 = = 31.3 nF
co R1
1
R3 = = 115 
co KC3
7-30)
 1  1
Ramp function gain = 20 log10   = 20 log10   = −9.54 dB
V  3
 p
Total gain = - 8 dB - 9.54 dB = -17.54 dB
17.54
G = 10 20
= 7.54
Using 45 for the phase margin,
 comp = 45 − (−140) = 185
2
  185 + 90  
K =  tan    = 6.61
  4 
co = 2 f co = 2 (15, 000) = 94, 248 rad / s
Let R1 = 1 k 
G R1
R2 = = 2.93 k 
K
K
C1 = = 9.31 nF
co R2
1
C2 = = 1.41 nF
co R2 K
K
C3 = = 27.3 nF
co R1
1
R3 = = 151 
co KC3

7-31)
Using Vs = 20 V ,
gain at 10 kHz = −9.16 dB  co = −133
1
gain of pwm = 20 log10   = −9.54 dB
3
Total gain = − 9.16 − 9.54 = −18.7 dB
18.7
G = 10 20 = 8.61
 comp = 45 − (−133) = 178
Using equations 7 - 104 and 7 - 112,
K = 5.55
Let R1 = 1 k , R2 = 5.55 k 
C1 = 10.3 nF
C2 = 1.85 nF
C3 = 37.5 nF
R3 = 180 

100

Magnitude
(10.000K,-9.1569)
0 Phase

(10.000K,-133.095)
-100

-200
1.0Hz 10Hz 100Hz 1.0KHz 10KHz 100KHz
DB(V(Output)) P(V(Output))
Frequency

CHAPTER 8 SOLUTIONS
4/24/10
8-1)
Load: I avg = 0, I rms = 10 A.
Switches: I avg = 5 A., I rms = I m D = 10 0.5 = 7.07 A.
Source : I avg = I rms = 10 A. (See Example 2-4)
8-2)
Vdc  1 − e −T /2  L 0.1 1 V 96
I max =  ;  − = = 0.02s.; T = ; dc = = 19.2 A.
R  1 + e −T /2  R 5 60 R 5
 0.341 
I max = 19.2   = 3.94 A.
 1.66) 
V
b) From Eq. 8 − 1: io (t ) = dc + Ae −t /
R
V
io (0) = 0  A = − dc
R
io (t ) = dc (1 − e − t / )
V
R
i (T / 2) = 19.2 (1 − e −1/2.4 ) = 6.54 A.

c) PSpice results are consistent with parts (a) and (b). The current waveform
reaches steady state after approximately 100 ms, corresponding to 5 time
constants.
8.0A
(8.3333m,6.5486)

(158.333m,3.9485)
4.0A

0A

-4.0A
0s 40ms 80ms 120ms 160ms 200ms
I(L)
Time

8-3)

Vdc 150 L 40 mH T 1 / 60
a) = = 7.5 A.;  = = = 2 ms; = = 4.167
R 20 R 20  2 4 ms
Using Eq (8 − 8),

 1 − e −4.167 
I max = 7.5  −4.167  = 7.271 A.
1+ e 
I min = − I max = −7.271 A.

Using Eq. (8-5),

7.5 − 14.8e −t /.002 for 0  t  8.33 ms



io = 
−7.5 + 14.8e − (t −1/120)/.002 for 8.33 ms  t  16.7 ms

b)
c) I peak = 7.271 A.

d ) Vmax = Vdc = 150 V .

8-4)
Vdc 125 L 25 mH T 1 / 60
a) = = 6.25 A.;  = = = 1.25 ms; = = 13.33
R 20 R 20  2 1.25 ms
Using Eq (8 − 8),

 1 − e −13.33 
I max = 6.25  −13.33 
= 6.25 A.
1+ e 
I min = − I max = −6.25 A.

Using Eq. (8-5),

6.25 − 12.5e −t /.00125 for 0  t  8.33 ms



io = 
−6.25 + 12.5e − (t −1/120)/.00125 for 8.33 ms  t  16.7 ms

b) Using the first half-period,


1/120

 ( 6.25 − 12.5e )
1 − t /.00125
2
I rms = dt = 5.45 A.
120 0

R = ( 5.25) 20 = 594 W .
2
c) P = I rms
2

P 594
Is = = = 4.75 A.
Vdc 125

8-5)
Z1 = 152 +  2 ( 400 )( 0.01)  = 29.3 
2
a)

( )
V1 = I1Z1 = 8 2 ( 29.3) = 331 V .

4Vdc  V1
V1 =  Vdc = = 260 V .
 4
4Vdc Vn I
Z n = R 2 + ( 2 400 L ) ;
2
b) Vn = ; In = ; I n ,rms = n
n Zn 2
n Vn Zn In,rms
1 331 29.3 8.0
3 110 77 1.02
5 66 127 0.37

1.022 + 0.37 2
THDI = = 0.136 = 13.6%
8.0
8-6)
a) Z1 = 2.52 +  2 (120 )( 0.025 )  = 31.3 
2

( )
V1 = I1Z1 = 2 2 ( 31.3) = 88.6 V .

4Vdc  V1
V1 =  Vdc = = 69.6 V .
 4
4Vdc Vn In
Z n = R 2 + ( 2 120 L ) ;
2
b) Vn = ; In = ; I n ,rms =
n Zn 2

n Vn Zn In,rms
1 88.6 31.3 2.0
3 29.5 61.8 0.34
5 17.7 97.5 0.13

0.342 + 0.132
THDI = = 0.185 = 18.5%
2.0

Using PSpice,
FOURIER COMPONENTS OF TRANSIENT RESPONSE I(L_L)

DC COMPONENT = -3.668708E-06

HARMONIC FREQUENCY FOURIER NORMALIZED PHASE NORMALIZED


NO (HZ) COMPONENT COMPONENT (DEG) PHASE (DEG)

1 1.200E+02 2.830E+00 1.000E+00 -3.716E+01 0.000E+00


2 2.400E+02 5.377E-06 1.900E-06 -1.203E+02 -4.594E+01
3 3.600E+02 4.778E-01 1.688E-01 -6.658E+01 4.490E+01
4 4.800E+02 3.589E-06 1.268E-06 -1.223E+02 2.629E+01
5 6.000E+02 1.818E-01 6.422E-02 -7.587E+01 1.099E+02
6 7.200E+02 2.858E-06 1.010E-06 -1.162E+02 1.068E+02
7 8.400E+02 9.427E-02 3.331E-02 -8.028E+01 1.798E+02
8 9.600E+02 2.523E-06 8.913E-07 -1.095E+02 1.878E+02
9 1.080E+03 5.743E-02 2.029E-02 -8.292E+01 2.515E+02

TOTAL HARMONIC DISTORTION = 1.847695E+01 PERCENT

8-7)
Using a restricted time interval of 33.33 ms to 50 ms to analyze steady-state current, the
peak value is 8.26 A and the rms value is 4.77 A. The THD from the output file is 32%.
20A

Peak
10A
(35.134m,8.2603) rms
(50.000m,4.7738)

i(t)
0A

-10A
30ms 35ms 40ms 45ms 50ms
I(R) RMS(I(R))
Time

voltage (100 V)
10

current

S1, S2 D3, D4
0
D1, D2

S3, S4

-10

30ms 35ms 40ms 45ms 50ms


I(R) V(A)/10 0
Time

8-8)
 4V 
a ) V1 =  dc  cos ( )
  

 V1   90 
 = cos −1  −1
 = cos   = 55.6
 4Vdc   4 (125 ) 
 4V  V I
Vn =  dc  cos ( n ) ; Z n = R + jn0 L ; I n = n ; I n ,rms = n
 n  Zn 2
n |Vn| Zn In,rms
1 90 12.5 5.08
3 51.6 24.7 1.5
5 4.43 39 0.08

1.52 + 0.082
THDI  = 0.29 = 29%
5.08

8-9)
4Vdc 4 ( 200 )
a) V1 = = = 255 V .
 
Z1 = R + j0 L = 10 + j 2 60 ( 0.035) = 16.6 

V1 255
I1 = = = 15.3 A.
Z1 16.6

15.3
I1,rms = = 10.9 A.
2
b) At 30 Hz ,

Z1 = 10 + j 2 30 ( 0.035 ) = 12.0 

V1 = I1Z1 = (15.3)(12.0 ) = 184 V .

 4V   V   184 
 cos ( )   = cos −1  1 −1
V1 =  dc  = cos   = 43.7
    4Vdc   4 ( 200 ) 

8-10)
α = 30°
Using the FFT function in Probe shows that voltages at frequencies at multiples of n = 3
are absent.
b) α = 15°
Using the FFT function in Probe shows that voltages at frequencies at multiples of n = 5
are absent.

8-11)
From Eq. (8-22),

90 90
= = = 12.86
n 7

Using the FFT function in Probe, the n = 7 harmonic is absent.


8-12)
Letting T = 360 seconds and taking advantage of half-wave symmetry,
2  
54 114 150
Vrms = 
 Vm2 dt +  Vm2 dt + V m dt 
2
360  30 66 126

1
Vrms = Vm ( 54 − 30 ) + (114 − 66 ) + (150 − 126 )  = 0.730Vm
180 

8-13)
The VPWL_FILE source is convenient for this simulation. A period of 360 seconds is
used, making each second equal to one degree. A transient simulation with a run time of
360 second and a maximum step size of 1m gives good results. The FFT of the Probe
output confirms that the 3rd and 5th harmonics and their multiples are eliminated.

0 0
30 0
30.01 1
54 1
54.01 0
66 0
66.01 1
114 1
114.01 0
126 0
126.01 1
150 1
150.01 0
210 0
210.01 -1
234 -1
234.01 0
246 0
246.01 -1
294 -1
294.01 0
306 0
306.01 -1
330 -1
330 0
360 0

8-14)
a)
4Vdc
b) Vm = cos ( n1 ) + cos ( n 2 ) + cos ( n 3 ) 
n 
Vdc = 48 V ; 1 = 15 ;  2 = 25 ;  3 = 55
n 1 3 5 7 9
Vn 149.5 0 -2.79 -3.04 -14.4

cos (1 ) + cos ( 2 ) + cos ( 3 )


c) M i = = 0.815
3

8-15)
To eliminate the third harmonic,
cos(31 ) + cos(3 2 ) + cos(3 3 ) = 0

Select two of the angles and solve for the third.


Examples:
α1 α2 α3 Mi
15 25 55 0.815
20 30 40 0.857
10 30 50 0.831
10 30 70 0.731

8-16)
This inverter is designed to eliminate harmonics n = 5, 7, 11, and 13. The normalized
coefficients through n = 17 are

n Vn/Vdc
1 4.4593
3 -0.8137
5 0.0057 ≈ 0
7 -0.0077 ≈ 0
9 -0.3810
11 0.0043 ≈ 0
13 -0.0078 ≈ 0
15 -0.0370
17 0.1725
The coefficients are not exactly zero for those harmonics because of rounding of the
angle values.

8-17)

8-18)
V1 = V1,rms 2 = 54 2 = 76.8 V .

V1 76.8
ma = = = 0.8
Vdc 96

Z n = R + jn0 L = 32 + jn 2 60 (.024 ) = 32 + jn9.05

From Table 8-3,


n Vn/Vdc Vn Zn In=Vn/Zn
1 0.8 76.8 33.3 2.30
mf 17 0.82 78.7 157 0.50
mf - 2 15 0.22 21.1 139 0.151
mf + 2 19 0.22 21.1 175 0.121

2 2 2
 0.50   0.151   0.121 
  +  + 
 2   2   2 
THD = = 0.23 = 23%
2.30
2

8-19)
V1 = V1,rms 2 = 160 2 = 226.3 V .

V1 226.3
ma = = = 0.9
Vdc 250

Z n = R + jn0 L = 20 + jn 2 60 (.050 ) = 20 + jn18.85

From Table 8-3,


n Vn/Vdc Vn Zn In=Vn/Zn
1 0.9 225 27.5 8.18
mf 31 0.71 178 585 0.305
mf - 2 29 0.27 67 547 0.122
mf + 2 33 0.27 67 622 0.108

2 2 2
 0.305   0.122   0.108 
  +  + 
 2   2   2 
THD = = 0.044 = 4.4%
8.18
2
8-20)
The circuit “Inverter Bipolar PWM Function” is suitable to verify the design results.
The parameters are modified to match the problem values.

Transient Analysis and Fourier Analysis are establish in the Simulation Setup menu:

The output file contains the THD of the load current, verifying that the THD is less than
10%.

TOTAL HARMONIC DISTORTION = 9.387011E+00 PERCENT

8-21)
Example solution:
Let ma = 0.9,

Vm 120 2
Vdc = = = 189 V .
ma 0.9

Using Table 8-3, at n = m f , Vmf = 0.71(189 ) = 134 V .

for THD  8%, I mf  0.08I1

V1 120 2 120 2
I1 = = = = 13.6 A.
Z1 10 + j 2 60 ( 0.020 ) 12.5

I mf  0.08 (13.6 ) = 1.09 A.

Vmf 134
Z mf  = = 123   m f 0 L
I mf 1.09

123 123
mf  = = 16.4
0 L 377 ( 0.020 )

Choose odd integer 19 or greater for m f .

8-22)
Example solution:
V1 = V1,rms 2 = 100 2 = 141 V .

V1 141
Let ma = 0.9  Vdc = = = 157 V .
ma 0.9

V1 V1 141
I1 = = = = 4.48 A.
Z1 R + j0 L 30 + j 377 ( 0.025)

I mf
THDI   0.10  I mf  0.1( 4.48) = 0.448 A.
I1

Vmf 0.71(157 )
Z mf =  = 249   m f 0 L
I mf 0.448

249
mf  = 26.4
377 ( 0.025)

Choose odd integer 29 or greater for m f .

8-23)
Use the bipolar PWM function circuit of Fig. 8-23a, and use the unipolar PWM
function circuit of Fig.8-26 with mf = 10. Use ma = 0.8 for V1 = 120 V from the 150-V
dc source.
The THD for bipolar, mf = 21, is 10.2 %, for bipolar mf = 41 is 5.2%, and for unipolar
mf = 10 is 5.9%.

Bipolar mf = 21:
Bipolar mf = 41:

Unipolar, mf = 10:
8-24)
2Vdc     2    500 
a) V1, L − N =  2 + cos   − cos   =   3 = 159 V .
3   
3  3    3 

V1 159 159
I1 = = = = 6.09 A.
Z1 25 + j 377 ( 0.020 ) 26.1

I1
I1,rms = = 4.31 A.
2

8-25)

Use Eq. (8-42) for Vn,L-N , Z n = R + jn 2 fL , I n = Vn , L - N / Z n , and I n ,rms = I n / 2.

For f = 25 Hz:
n VnL-N Zn In In,rms
1 255 11.1 23.0 16.3
5 50.9 25.6 2.0 1.41
7 36.4 34.5 1.06 0.75
11 23.1 52.8 0.44 0.31
13 19.6 62.0 0.32 0.22

1.412 + 0.752 + 0.312 + 0.222


THDI = = 0.10 = 10%
16.3
50.92 + 36.42 + 0.222 + 19.62
THDV = = 0.273 = 27.3%
255

For f = 100 Hz,


n VnL-N Zn In In,rms
1 255 21.3 11.9 8.43
5 50.9 94.8 0.54 0.38
7 36.4 132 0.27 0.19
11 23.1 208 0.12 0.08
13 19.6 245 0.08 0.06

0.382 + 0.192 + 0.082 + 0.062


THDI = = 0.0519 = 5.19%
8.43

The THD for current is reduced from 10% to 5.19% as f is increased from 25 Hz to 100
Hz. The THD of the line-to-neutral voltage remains at 27.3%.

These results can also be determined from a PSpice simulation for the six-step inverter.
8-26)
( )
At f = 30 Hz , Z1 = 10.7 , V1 = I1Z1 = 10 2 (10.7 ) = 151 V .

2Vdc     2  
V1, L − N =
3 
 2 + cos   − cos    = Vdc ( 0.637 )
3  3 

V1, L − N 151
Vdc = = = 237 V .
0.637 0.637

( )
At f = 60 Hz , Z1 = 19.5 , V1 = I1Z1 = 10 2 (19.5 ) = 276 V

276
Vdc = = 433 V .
0.637

CHAPTER 9 SOLUTIONS
3/13/10

9-1)
1
0 = = 1.83(10)6 rad / s
LC
L
Z0 = = 1.83
C
I L
t1 = 0 r = 0.5  s
V3
1  −1  I 0 Z 0  
t2 − t1 = sin   +   = 2.35  s
0   Vs  
C V (1 − cos 0 (t2 − t1 ))
t3 − t 2 = r s = 0.845  s
I0
t 
V0 = Vs f s  1 + (t2 − t1 ) + (t3 − t2 )  = 5.17 V .
2 
9-2)
1
0 = = 1.69(10)6 rad / s
LC
L
Z0 = = 0.845
C
I L
t1 = 0 r = 0.083  s
Vs
1  −1  I 0 Z 0  
t2 − t1 = sin     = 1.94  s
0   Vs  
C V (1 − cos 0 (t2 − t1 ))
t3 − t 2 = r s = 8.36  s
I0
f max = 1/ t3 = 1/ (0.083 + 1.94 + 8.36)  s = 96.3 kHz
t 
Vo = Vs f s  1 + ( t2 − t1 ) + ( t3 − t2 )  = 17.9 V
2 
for Vo = 5V ,
V0
fs =  26.9 kHz
t 
Vs  1 + (t2 − t1 ) + (t3 − t2 ) 
2 
9-3)
1
a ) 0 = = 108
LC
L
Z0 = =1
C
I L
t1 = 0 r = 1.39 ns
Vs
1  −1  I 0 Z 0  
t2 − t1 = sin   +   = 32.8 ns
0   Vs  
C V (1 − cos 0 (t2 − t1 ))
t3 − t 2 = r s = 143.3 ns
I0
t 
Vo = Vs f s  1 + (t2 − t1 ) + (t3 − t2 )  = 4.77 V .
2 
V 36
b) I L , peak = I o + s = 5 + = 41 A.
Z0 1
VC , peak = 2Vs = 72 V .
 12 
c) f s = 750 kHz    1.89 MHz
 4.77 
9-4)
Vs 50
I L , peak = I o + = 3+ = 9 A.  Z 0 = 8.33 
Z0 Z0
1 1
0 =  C=
LC L02
L L
Z0 = = = L0
C 1/ L02
Z0 8.33
L= = = 0.833  H
0 107
1
C= = 12 nF
L02
I 0 Lr
t1 = = 50 ns
Vs
1  −1  I 0 Z 0  
t2 − t1 = sin   +   = 366 ns
0   Vs  
C V (1 − cos 0 (t2 − t1 ))
t3 − t 2 = r s = 373 ns
I0
Vo
fs = = 945 kHz
t 
Vs  1 + (t2 − t1 ) + (t3 − t2 ) 
2 
9-5)
For I o = 0.5 A.
f 0 = 503 kHz
t1 = 0.05  s
t2 − t1 = 1.04  s
t3 − t2 = 3.97  s
Vo
fs = = 99.2 kHz
t 
Vs  1 + (t2 − t1 ) + (t3 − t2 ) 
2 

For I o = 3 a.
t1 = 0.30  s
t2 − t1 = 1.388  s
t3 − t2 = 0.439  s
Vo
fs = = 253 kHz
t 
Vs  1 + (t2 − t1 ) + (t3 − t2 ) 
2 

 99.2 kHz  f s  253 kHz


9-6)
RL 5 V 15
= = 2; o = = 0.5
Z 0 2.5 Vs 30
fs  2 (200)(10)3
From Fig. 9 − 1g ,  0.27  0 = s = = 4.65(10) 6 rad / s
f0 0.27 0.27
1 1
0 =  LC = 2
LC 0
L
Z0 =  L = Z 02C
C
1 1 1
LC = Z 02C 2 = 2  C = = = 0.086  F
0 Z 0 0 (2.5)(4.65)(10)6
1
L= = 0.538  H
02C

9-7) a) The circuit is shown with diode D2 added to make the switch unidirectional.
20
V(C)

Average voltage
(6.6839u,5.1605)
I(L)
10

-10
0s 2.0us 4.0us 6.0us 8.0us 10.0us 12.0us
V(D1:2) I(L1) V(S1:1) AVG(V(D1:2))
Time

(a) The average output (capacitor) voltage is 5.16 V, agreeing with the 5.17 V computed
analytically. (b) Peak capacitor voltage= 20 V.; (c) Inductor currents: peak = 10.5 A.;
average = 2.59 A.; rms = 4.54 A.
9-8)

  t 
Vo = Vs 1 − f s  t3 − 1  
  2 
1 − Vo / Vs 1 − 15 / 20
fs = = = 182 kHz
t3 − t1 / 2 1.46(10) −6 − 0.188(10) −6
9-9)
1 1
0 = = −8
= (10)8 rad / s
Lr Cr (10)
Lr
Z0 = = 10 
Cr
Vs Cr
t1 = = 2 ns
Io
1  −1  Vs  
t2 = sin   +   + t1 = 35.4 ns
0   Io Z0  
LI
t3 = r o [1 − cos 0 (t2 − t1 )] + t2 = 134.4  s
0
  t 
Vo = Vs 1 − f s  t3 − 1   = 14.7 V .
  2 
I L , peak = I o = 10 A.
L
VC , peak = Vs + I o = 114.7 V .
C
9-10)
1
0 = = (10)7 rad / s
Lr Cr
Lr
Z0 = = 10 
Cr
Vs Cr
t1 = = 16.7 ns
Io
1  −1  Vs  
t2 = sin   +   + t1 = 348 ns
0   Io Z0  
LI
t3 = r o [1 − cos 0 (t2 − t1 )] + t2 = 1.54  s
0
  t 
Vo = Vs 1 − f s  t3 − 1   = 1.17 V .
  2 
1 − Vo / Vs
For Vo = 2.5, f s = = 326 kHz.
t3 − t1 / 2
9-11)
1
0 = = 1.414(10)7 rad / s
Lr Cr
Lr
Z0 = = 7.07 
Cr
Vs Cr
t1 = = 12 ns
Io
1  −1  Vs  
t2 = sin   +   + t1 = 246 ns
0   Io Z0  
LI
t3 = r o [1 − cos 0 (t2 − t1 )] + t2 = 1.07  s
0
  t 
Vo = Vs 1 − f s  t3 − 1   = 5.6 V .
  2 
For I o = 8 A., t1 = 15 ns, t2 = 252 ns, t3 = 911 ns
1 − Vo / Vs
fs = = 394.1 kHz.
t3 − t1 / 2
For I o = 15 A., t1 = 8 ns, t2 = 238 ns, t3 = 1.48  s
f s = 645.4 kHz

394.1 kHz  f s  645.4 kHz


9-12)
VC , peak − Vs 40 − 15
VC , peak = Vs + I o Z 0  Z 0 = = = 6.25 
Io 4
Lr
Z0 =  Lr = Z 02C
Cr
1 1
0 =  Cr =
Lr Cr Lr02
 1  Z 6.25
Lr = Z 02Cr = Z 02  2 
 Lr = 0 = = 3.91  H
 Lr0  0 1.6(10)6
1
C= = 0.1  F .
Lr02
Vs Cr
t1 = = 0.375  s
Io
1  −1  Vs  
t2 = sin   +   + t1 = 2.74  s
0   Io Z0  
LI
t3 = r o [1 − cos 0 (t2 − t1 )] + t2 = 4.62  s
Vs
1 − Vo / Vs 1 − 5 /15
fs = = = 150 kHz.
t3 − t1 / 2 (4.62 − 0.375 / 2)(10) −6
9-13)
Vo 15
Io = = = 3 A.
RL 5
RL 5 V 15
= = 0.2; o = = 0.5
Z 0 25 Vs 30
fs f 100 kHz
From Fig. 9 − 2 g ,  0.37  f 0 = s = = 270 kHz.
f0 0.37 0.37
1
0 = 2 f 0 = 1.70(10)6 rad / s =
Lr Cr
Lr Z 25
Z0 =  Lr = 0 = = 14.7  H
Cr 0 1.70(10)6
Lr
Cr = = 23.5 nF .
Z 02

9-14) A suitable circuit is shown. The values of the output filter components L1 and C2
are not critical. The load resistor is chosen to give 10 A. The switch must be open for an
interval between t2 and t3; 50 ns is chosen.
400u

Energy from source per period


(149.500u,-72.676u)
0

SEL>>
-400u
S(W(V1))

(149.470u,120.125)
100V
Capacitor

50V Output
149.088u,14.578)

0V
149.0us 149.2us 149.4us 149.6us 149.8us 150.0us
V(INPUT,D3:2) V(R1:1)
Time

Results from Probe for steady-state output: a) Vo ≈ 14.6 V., b) VC,peak= 120 V., c)
Integrate instantaneous power, giving 72.7 μJ per period (supplied).

9-15)
4Vdc
V1 = 80 2 = 113 V . =  Vdc = 88.9 V .

V3
THD   V3  (0.05)(113) = 5.66 V .
V1
V1 113
For a square wave, V3 = = = 37.7 V (input to filter )
3 3
Vo ,3 5.66 1
= = 2
 Q = 2.47
2  30 0 
Vi ,3 37.7
1+ Q  − 
 0 30 
1
C= = 13.4  F .
Q0 R
1
L= = 11.8 mH .
02C
V1 V 113
VC , peak = = 280 V .; I L , peak = 1 = = 9.43 A.
0 RC R 12
9-16)
4Vdc
V1 = 100 2 = 141 V . =  Vdc = 111 V .

V3
THD   V3  (0.1)(141) = 14.1 V .
V1
V1 141
For a square wave, V3 = = = 47 V (input to filter )
3 3
Vo ,3 14.1 1
= = 2
 Q = 1.19
Vi ,3 47  3  
1 + Q2  0 − 0 
 0 30 
1
C= = 13.9  F .
Q0 R
1
L= = 1.27 mH .
02C
200

100

(10.416m,3.6212)
0

-100

-200
10.0ms 10.4ms 10.8ms 11.2ms 11.6ms 12.0ms
V(V1:+) V(OUT) I(R1)
Time

The output file shows that the THD is 10.7%. Increase Q by increasing L, and adjust C
accordingly. L=1.4 mH and C=12.6 µF gives THD=9.8%. Switching takes place when
load (and switch) current is approximately 3.6 A.

9-17)
V1,2rma
P  V1,rms = PR = 500(15) = 86.6 V .
R
4Vdc
V1 = 86.6 2 = 122.5 V . =  Vdc = 96.2 V .

V3
THD   V3  (0.1)(122.5) = 12.25 V .
V1
V1 122.5
For a square wave, V3 = = = 40.8 V (input to filter )
3 3
Vo ,3 12.25 1
= = 2
 Q = 1.19
Vi ,3 40.8  3  
1 + Q2  0 − 0 
 0 30 
1
C= = 17.8  F .
Q0 R
1
L= = 5.68 mH .
02C
V1 122.5
VC , peak = = = 146 V .
0 RC (2 500)(15)(17.8)(10) −6
V1 122.5
I L , peak = = = 8.17 A.
R 15
200

v(out)
V(cap)
100 V(in)

(10.434m,8.1206)
I(L)
0

-100

-200
10.0ms 10.4ms 10.8ms 11.2ms 11.6ms 12.0ms
V(OUT) V(IN) V(L1:2,C1:2) I(L1)
Time

From the output file, THD = 10.8%. From Probe: VC,peak=149 V.; IL,peak=8.12 A.

9-18)
2
f 0 = 20 = = 839 kHz  f3  f 0
Lr Cr
8 RL
Re = = 8.11 
2
s = 2 f s = 5.65(10)6
X L = s Lr = 33.9 
1
XC = = 29.5 
 s Cr
   
   
Vs  1  10  1  = 4.38 V .
Vo =  2 
=
 −  
2
 1+  X L − XC   
2 2 33.9 29.5
   1+   
    8.11  
  Re  

9-19)
2
f 0 = 20 = = 1.33 MHz  f s  f 0
Lr Cr
8 RL
Rs = = 4.05 
2
s = 2 f s = 9.42(10)6 rad / s.
X L = s Lr = 11.3 
1
XC = = 8.84 
 s Cr
 
 
Vs  1 
Vo =  2 
= 10.25 V .
2
 1+  X − X  
  R  
L C

  s  
9-20)
Vo 6
= = 0.3
Vs 18
s
Let = 1.2  Q  3 from Fig. 9 − 5d
0
s s 2 (800, 000)
0 = = = = 4.19(10) rad / s
6

1.2 1.2 1.2


QRL 3(5)
Lr = = = 3.58  H
0 4.19(10)6
1
Cr = = 1.59(10) −8 = 15.9 nF .
 Lr 2
0
A PSpice simulation using the circuit of Fig. 9-6(a) gives an output voltage of
approximately 5.1 V.

9-21)
Vo 18
= = 0.36
Vs 50
s
Let = 1.2  Q  2.1 from Fig . 9 − 5d
0
s s
2 (10)6
0 = = = = 5.23(10)6 rad / s
1.2 1.2 1.2
QRL 2.1(9)
Lr = = = 3.61  H
0 5.23(10)6
1
Cr = = 10.1 nF .
 Lr
2
0

9-22)

Vo 15
= = 0.375
Vs 40
s
Let = 1.2  Q  1.9 from Fig. 9 − 5d
0
s 2 (800, 000)
0 = = = 4.19(10)6 rad / s
1.2 1.2
QRL 1.9(5)
Lr = = = 2.27  H
0 4.19(10)6
1
Cr = 2 = 25.1 nF .
0 Lr

A PSpice simulation using the circuit of Fig. 9-6a shows a steady-state output
voltage of approximately 14.4 V, slightly less than the target value of 15 V. Note
that the current in Lr and Cr is not quite sinusoidal.

9-23)
Vo 55
= = 0.367
Vs 150
s
Let = 1.2  Q  2 from Fig . 9 − 5d
0
s 2 (1, 000, 000 )
If f s = 1 MHz , 0 = = = 5.23(10)6 rad / s
1.2 1.2
QRL 2(20)
Lr = = = 7.64  H
0 5.23(10)6
1
Cr = = 4.77 nF .
 Lr
2
0

A PSpice simulation using the circuit of Fig. 9-6a shows a steady-state output voltage of
approximately 53 V, slightly less than the target value of 55 V. Note that the current in
Lr and Cr is not quite sinusoidal.

9-24)

RL 2 10 2
Re = = = 12.3 
8 8
1
0 = = 2.53(10)6 rad / s
Lr Cr
0
f0 = = 403 kHz.  f s  f 0
2
X L = s Lr = 4.08 
1
XC = = 2.65 
 s Cr
4Vs
Vo = = 9.60 V .
2 2
 XL   XL 
 2 1 −  + 
 X C   Re 
9-25)

RL 2 15 2
Re = = = 18.5 
8 8
1
0 = = 5.66(10)6 rad / s
Lr Cr
0
f0 = = 901 kHz.  f s  f 0
2
X L = s Lr = 7.54 
1
XC = = 6.12 
 s Cr
4Vs
Vo = = 25.9 
2 2
 XL   XL 
 2 1 −  + 
 X C   Re 
9-26)
Vo 20
= = 1.67
Vs 12
f s s  2 500000
Let = = 1.05  0 = s = = 2.99(10)6 rad / s
f 0 0 1.05 1.05
From Fig . 9 − 10c, Q  3.8
R 15
Lr = L = = 1.32  H
0Q 2.99(10)6 (3.8)
1
Cr = = 84.7 nF
 Lr
2
0
9-27)

Vo 36
= = 0.8
Vs 45
f s s  2 900000
Let = = 1.1  0 = s = = 5.14(10)6 rad / s
f 0 0 1.1 1.1
From Fig. 9 − 10c, Q  1.9
R 20
Lr = L = = 2.05  H
0Q 5.14(10)6 (1.9)
1
Cr = = 18.5 nF
 Lr
2
0

9-28)
Vo 60
= = 1.2 Let f s = 500 kHz
Vs 50
f s s  2 500000
= = 1.1  0 = s = = 2.86 (10 ) rad / s
6
Let
f 0 0 1.1 1.1
From Fig . 9 − 10c, Q  3.4
R 25
Lr = L = = 2.57  H
0Q 2.86(10)6 (3.4)
1
Cr = = 47.6 nF
 Lr
2
0

9-29)
Vs = 100 V .; f s = 500 kHz; RL = 10 ; L = 12  H ; Cs = C p = 12  F
2 2
Re = RL = (10) = 12.3 
8 8
s = 2 f s = 2 500000 = 3.14(10)6
4Vs
Vo = = 37.7 V .
2 2
 Cp   L 1 
2 1 + − s2 LC p  +  s − 
 Cs   Re s ReCs 

9-30)
Sample solution
Vo 5
= = 0.417
Vs 12
f s s
Let = = 1.2  Q  2.7 from Fig . 9 − 11c
f 0 0
s2 800000
0 = = = 4.19 (10 ) rad / s
6

1.2 1.2
QRL 2.7(2)
L= = = 1.29  H .
0 4.19(10)6
1
Cs = C p = = 44.2 nF .
02 L
Check with Eq. 9 − 74 : Vo = 4.97 V .
9-31)
Sample solution
Vo 5 V 5
= = 0.25; RL = o = = 5 
Vs 20 Io 1
f s s
Let = = 1.2  Q  5 from Fig . 9 − 11c
f 0 0
s2 750000
0 = = = 3.93(10)6 rad / s
1.2 1.2
QRL 5(5)
L= = = 6.37  H .
0 3.93(10)6
1
Cs = C p = = 10.2 nF .
02 L
Check with Eq. 9 − 74 : Vo = 5.08 V .
9-32)
Sample solution
Vo 10 V 10
= = 0.4; RL = o = = 10  ; Let f s = 100 kHz
Vs 25 Io 1
f s s
Let = = 1.15  Q  3.3 from Fig . 9 − 11c
f 0 0
s 2 100000
0 = = = 546 krad / s
1.15 1.15
QRL 3.3(10)
L= = = 60.4  H .
0 5.46(10)5
1
Cs = C p = = 55.5 nF .
02 L
Check with Eq. 9 − 74 : Vo = 10.02 V .

Using a circuit based on Fig. 9-11a but with a square-wave source implemented
with Vpulse (see Fig. 9-6a), the result is approximately 9.4 V.

9-33)
(a) A PSpice simulation using the circuit shown reveals that the capacitor
voltage returns to zero at 15.32 μs and the switch must remain closed for 5.58 μs
for the inductor current to return to 12 A. Initial conditions for the inductor (12
A) and for the capacitor (0 V) must be applied. Ideal models for the switch and
diode are used.
b) Using the expression S(W(V1)) for energy (S is integration), 15.7 mJ are
supplied by the voltage source in one period.
c) Average power is 754 W, obtained by entering AVG(W(V1)).
d) Average resistor power is 104 W.
e) With R = 0, the capacitor voltage returns to zero at 15.44 μs and the switch
must remain closed for 5.45 μs. The source power and energy are not changed
significantly.
20 (20.901u,12.000)
I(L)
15.318u,16.390m)
0

-20

14.71us 16.00us 18.00us 20.00us 21.53us


V(switch) V(cap) I(L1)
Time

200
V(capacitor)

SEL>>
-200
0s 4us 8us 12us 16us 20us 24us
V(SWITCH) V(CAP) I(L1)
Time
9-34)
1 1
o = = = 2(10)5 rad / s
−6 −6
Lr Cr 250(10) 0.1(10)
R 1
= = = 2000
2 Lr 2(250)(10)−6
 = o2 −  2  0
vC (t )  Vs + e − t [−Vs cos ot + o Lr ( I1 − I o ) sin ot ]
= 75 + e−2000t [−75cos(2(10)5 t ) + 2(10)5 250(10)−6 (7 − 5) sin(2(10)5 t )]
= 75 + e−2000t [−75cos(2(10)5 t ) + 100sin(2(10)5 t )]
vC (t x ) = 0  t x = 22.3  s
 V 
iL ( t )  I o + e − t ( I1 − I o ) cos ot + s sin ot 
 o Lr 
 75 
= 5 + e −2000t (7 − 5(cos(2(10)5 t ) + 5 −6
sin(2(10)5 t ) 
 2(10) 250(10) 
= 5 + e −2000t  2 cos(2(10)5 t ) + 1.5sin(2(10)5 t ) 
iL (t x ) = iL (22.3  s ) = 3.14 A.
iL Lr (7 − 3.14)250(10)−6
t = = = 12.9  s
Vs 75
9-35)
1 1
o = = 3.65(10)5 rad / s
−6 −6
Lr Cr 150(10) 0.05(10)
R 0.5
= = = 1667
2 Lr 2(150)(10)−6
 = o2 −  2  0

CHAPTER 10 SOLUTIONS
3/20/10

10-1)
a) For the elementary MOSFET drive circuit, losses can be determined from
the energy absorbed by the transistor. In Probe, the integral of instantaneous
power is obtained by entering the expression S(W(M1)) to get the energy
absorbed by the transistor. For turn-off losses, restrict the data to 2.5 µs to 4.3
µs. The energy absorbed is 132 µJ. For turn-on losses, restrict the data to 5 µs
to 5.6 µs. The energy absorbed by the MOSFET is 53.3 µJ. Power is
determined as
1 1
T= = = 5 s
f s 20000
W 132  J
Pturn −off = = = 26.4 W .
T 5 s
W 53.3  J
Pturn −on = = = 10.7 W .
T 5 s

For the emitter-follower drive circuit restrict the data to 2.5 µs to 2.9 µs, giving
21.3 µJ for turn-off. Restrict the data to 5 µs to 5.3 µs, giving 12.8 µJ for turn-
on. Power is then
W 21.3  J
Pturn −off = = = 4.26 W .
T 5 s
W 12.8  J
Pturn −on = = = 2.56 W .
T 5 s

b) For the first circuit, peak gate current is 127 mA, average gate current is zero,
and rms gate current is 48.5 mA. For the second circuit, peak gate current is 402
mA (and -837 mA), average gate current is zero, and rms gate current is 109
mA.

10-2)
For R1 = 75 Ω, toff ≈ 1.2 μs, ton ≈ 0.6 μs, and PMOS ≈ 30 W.
For R1 = 50 Ω, toff ≈ 0.88 μs, ton ≈ 0.42 μs, and PMOS ≈ 22 W.
For R1 = 25 Ω, toff ≈ 0.54 μs, ton ≈ 0.24 μs, and PMOS ≈ 14 W.

Reducing drive circuit resistance significantly reduces the switching time and
power loss for the MOSFET.

10-3)
The values of Vi, R1, R2, and C must be selected for the BJT base drive circuit.
First, select Vi: let Vi=20 V. then, the value of R1 is determined from the initial
current spike requirement. Solving for R1 in Eq. 10-1,
Vi − vBE 20 − 1
R1 = = = 3.8 
I B1 5

The steady-state base current in the on state determines R2. From Eq. 10-2,
Vi − vBE 20 − 1
R2 = − R1 = − 3.8 = 34.2 
IB2 0.5
The value of C is determined from the required time constant. For a 50% duty
ratio at 100 kHz, the transistor is on for 5 µs. Letting the on time for the
transistor be five time constants, τ = 1µs. From Eq. 10-3,

 R1 R2 
 = RE C =   C = (3.42)C = 1  s
 R1 + R2 
C = 0.292  F
10-4)
The values of Vi, R1, R2, and C must be selected for the BJT base drive circuit.
First, select Vi; let Vi = 20 V. then, the value of R1 is determined from the
initial current spike requirement. Solving for R1 in Eq. 10-1,
Vi − vBE 20 − 1
R1 = = = 6.33 
I B1 3

The steady-state base current in the on state determines R2. From Eq. 10-2,
Vi − vBE 20 − 1
R2 = − R1 = − 6.33 = 25.3 
I B2 0.6

The value of C is determined from the required time constant. For a 50% duty
ratio at 120 kHz, the transistor is on for 4.17 µs. Letting the on time for the
transistor be five time constants,
τ = 1 µs. From Eq. 10-3,

 R1 R2 
 = RE C =   C = 5.06 C = 0.833  s
 R1 + R2 
C = 0.165  F
10-5)
a) From Eq. 10-5 through 10-7 for t < tf,

 t   t 
iQ = I L 1 −  = 4 1 − −6 
= 4 − 8(10) 6 t
 t   0.5(10) 
 f 

I t 4t
iC = L = = 8(10)6 t
tf 0.5(10) −6
I Lt 2 4t 2
vC (t ) = = −6 −6
= 8(10)13 t 2
2Ct f 2(0.05)(10) (0.5)(10)

For tf < t < tx,


iQ = 0
iC = I L = 4
IL I t
vC = (t − t f ) + L f = 8(10)7 (t − 0.5(10) −6 ) + 20
C 2C

Time tx is defined as when the capacitor voltage reaches Vs (50 V.):

vC (t x ) = Vs = 50 = 8(10) 7 (t x − 0.5(10) 6 ) + 20  t x = 0.875 s


b) With tx > tf, the waveforms are like those in Fig. 10.12(d).
c) Turn-off loss is the switch is determined from Eq. 10-12,

42 [0.5(10)−6 ]2 (120000)
I L2t 2f f
PQ = = = 0.4 W .
24C 24(0.05)(10)−6
Snubber loss is determined by the amount of stored energy in the capacitor that
will be transferred to the snubber resistor:

1 0.05(10)−6 (50)2 (120000)


PR = CVs f =
2
= 7.5 W .
2 2
10-6)
Switch current is expressed as

 t   t 
iQ = I L 1 −  = 4 1 − −6 
= 4 − 8(10)6 t
 t   0.5(10) 
 f 

I t 4t
iC = L = = 8(10)6 t
tf 0.5(10) −6
I Lt 2 4t 2
vC (t ) = = −6 −6
= 4(10)14 t 2
2Ct f 2(0.01)(10) (0.5)(10)
Capacitor voltage at t = tf = 0.5 µs would be 100 volts, which is greater than Vs.
Therefore, the above equations are valid only until vC reaches Vs:

vC (t x ) = Vs = 50 = 4(10)14 t x2  t x = 0.354  s
For tx < t < tf,

iQ = 4 − 8(10) 6 t
iC = 0
vC = Vs = 50

b) With tx < tf, the waveforms are like those of Fig. 10.12(b).
Equation 10-12 is not valid here because tx < tf. Switch power is determined from
T T T
1 1
PQ =  p(t )dt =  iQ vQ dt = f  iQ vC dt =
T 0 T 0 0

tx tf

120000    4 − 8 (10 ) t   4(10) t  dt +  4 − 8 (106 ) t  (50) dt  = 1.84 W .
 6
 14 2

 0 tx 
Snubber loss is determined by the amount of stored energy in the capacitor that
will be transferred to the snubber resistor:

1 0.01(10−6 (50) 2 (120000)


PR = CVs2 f = = 1.5 W .
2 2
10-7)
I Lt f 10(0.1)(10) −6
C= = = 3.33 nF .
2Vs 2(150)
ton D / f 0.4 /100000
R= = = = 240 
5C 5C (5) 3.33(10) −9
1
PR = (3.33(10) −9 (150) 2100000 = 3.75 W .
2
I L2t 2f f 102 [0.1(10) −6 ]2105
PQ = = = 1.25 W .
24C 24(3.33)(10) −9
10-8)
I Lt f 10(0.1)(10) −6
C= = = 6.67 nF .
2V f 2(75)
ton D / f 0.4 /100000
R= = = = 120 
5C 5C 5(6.67)(10) −9
1 1
PR = CVs2 f = (6.67)(10)9 (150) 2100000 = 7.5 W .
2 2
I L t f f 102 [.1(10) −6 ]2105
2

PQ = = = 0.625 W .
24C 24(6.67)(10) −7
10-9)

7(0.5)(10) −6
I Lt f
C= = = 10.3 nF .
2Vs 2(170)
ton D / f 0.4 /125000
R= = = = 62.2 
5C 5C 5(10.3)(10) −9
1 1
PR = CVs2 f = (10.3)(10) −9 (170) 2125000 = 18.6 W .
2 2
I t f 102 [0.5(10) −6 ]2125000
2 2

PQ = L f = = 6.2 W .
24C 24(10.3)(10)−9
10-10)
I Lt f 7(0.5)(10) −6
C= = = 14.0 nF .
2V f 2(125)
ton D / f 0.4 /125000
R= = = = 45.7 
5C 5C 5(14)(10) −9
1 1
PR = CVs2 f = (14)(10) −9 (170) 2125000 = 25.3 W .
2 2
I t f 102 [0.5(10) −6 ]2125000
2 2

PQ = L f = = 4.56 W .
24C 24(14)(10) −9
10-11)
Using the snubber circuit of Fig. 10-12(a), Eq. 10-12 yields

I L2t 2f f 52 [0.5(10) −6 ]2 200000


C= = = 52.1 nF .
24 PQ 24(1)
ton D / f 0.35 / 200000
R= = = = 6.72 
5C 5C 5(52.1)(10) −9
1 1
PR = CVs2 f = (52.1)(10) −9 (80) 2 200000 = 33.3 W .
2 2
10-12)
Using the snubber circuit of Fig. 10-12(a), Eq. 10-12 yields

I L2t 2f f 62 [1(10) −6 ]2100000


C= = = 75 nF .
24 PQ 24(2)
ton D / f 0.3 /100000
R= = = = 8.0 
5C 5C 5(75)(10) −9
1 1
PR = CVs2 f = (75)(10) −9 (120) 2100000 = 54 W .
2 2

10-13)
a) From Eq. (10-16), TJ = TA + P ( R , JA ) = 30 + 2 ( 40 ) = 110 C

TJ − TA 150 − 30
b) P = = =3W
R , JA 40
10-14)

a) From Eq. (10-16), TJ = TA + P ( R , JA ) = 25 + 1.5 ( 55) = 107.5 C

TJ − TA 175 − 25
b) P = = = 2.73 W
R , JA 55

10-15)

TJ = P ( R , JC + R ,CS + R , SA ) + TA = 10 (1.1 + 0.9 + 2.5 ) + 40 = 85 C


10-16)

TJ = P ( R , JC + R ,CS + R , SA ) + TA = 5 (1.5 + 1.2 + 3.0 ) + 25 = 53.5 C


10-17)

TJ = P ( R , JC + R ,CS + R , SA ) + TA

TJ − TA 110 − 40
R , SA = − R , JC − R ,CS = − 0.7 − 1.0 = 2.19 C / W
P 18

10-18)
From Fig. 10.24 using the bottom curve for a single pulse,
Z , JC  0.013 C / W for a pulse of 10 −5 sec.
TJ = Pdm Z , JC = ( 500 W )( 0.013 C / W ) = 6.5 C

10-19)
a) For 50 kHz and D = 0.1, the pulse width is 2s. From Fig. 10.24, Z , JC  0.11C / W .
TJ = Pdm Z , JC = (100 )( 0.11) = 11 C

b) Using R  , JC = 1.05 C / W ,
TJ = Pavg R , JC = ( Pdm D ) R , JC = 100 ( 0.1)1.05 = 10.5 C.

Note that the value of Z from the graph is very rough,


and more precise evaluation in (a) is closer to the 10.5 of part (b).

You might also like